You are on page 1of 104

SCHOOL OF ENGINEERING AND ARCHITECTURE

ARPHYS1 PHYSICS FOR ARCHITECTS


(2nd Sem SY 2022-2023)

Prepared by:
A Self-regulated Learning Module
MICHELLE S. WALLANG, REE, MA-PHYS

A Self-regulated Learning Module 1


INTRODUCTION OF THE MODULE 3

MODULE 1: KINEMATICS 8

MODULE 2: DYNAMICS: LAWS OF MOTION 22

MODULE 3: ANGULAR MOTION 27

MODULE 4: EQUILIBRIUM 30

MODULE 5: WORK, POWER AND ENERGY 36

MODULE 6: IMPULSE AND MOMENTUM 40

MODULE 7: ELASTICITY 43

MODULE 8: FLUIDS 48

MODULE 9: HEAT TRANSFER 52

MODULE 10: OSCILLATIONS AND WAVE MOTION 54

MODULE 11: ELECETROSTATICS 62

MODULE 12: ELECTRICITY 67

MODULE 13: MAGNETISM 84

MODULE 14: LIGHT AND OPTICS 92

REFERENCES 100

EVALUATION OF THE COURSE 103

A Self-regulated Learning Module 2


1. Course Code: ARPHYS1
Course Title: PHYSICS FOR ARCHITECTS

2. Course Description

Physics 1 course covers vectors; kinematics; dynamics; work, energy and power;
impulse and momentum; rotation; elasticity; oscillation; fluids; thermal expansion;
thermal stress; heat transfer; calorimetry; waves; electrostatics; electricity and
magnetism; optics; image formation by plane and spherical mirrors.

3. Requirements of the Course

a. Physics 1 is a problem-solving course, thus, the measure of a student’s progress


is demonstrated by his ability to solve algebraic and trigonometric problems, aside
from knowing and reciting facts, laws and formulas.

b. The assigned activities will help you develop these skills and the exams will test
you on these skills. Activities that involve qualitative analysis and quantitative
calculations involving the topics discussed are included at the end of each lesson.
The following are used to measure the extent of the knowledge you gained during
the conduct of this course, and is given after each lesson:

 Activity – could be an assignment, quiz, seatwork or research work


 Assessment – given at the end of the module
 All course materials, such as syllabus, modules, etc., shall be
uploaded in CANVAS.
 Submission of all activities shall be in the form of hard copy, during
our face-to-face meeting. None will be submitted thru canvas, email
nor messenger.
 Activities must be submitted on time.
 For late submission, please note of the following:
1. if excuse: submit supporting documents and submit missed
activity upon entering the class, incase of assignment. For
quizzes and seat works, it must be taken within 48 hrs from the
time the activity was given.
2. If unexcused:
*Attendance: Asynchronous- reactions in class gc

F2f – actual checking, Affidavit of undertaking is required

- After 3 absences, names will be dropped sa OSA, readmission is needed for


you to be able to attend the class
- 3 lates are equivalent to 1 absence

A Self-regulated Learning Module 3


 NOTE: All activities are HAND-WRITTEN, except if there are illustrations or drawings
required which can be printed.

c. It is expected that you have an adequate working knowledge of algebra and


trigonometry. In order to do all the calculations necessary for this course, you are
required to have a scientific calculator at hand.

d. When instructed to write your solutions for a given set of problems, please use
short bond paper with the following format:

 The margin for the boarders is 1 cm all around and is drawn using a red ballpen.
 The heading’s height is 1 inch, then divided into half as shown. A 1in 2 box should be
drawn at the top rightmost corner. This is where your score for the activity will be placed.
 Write your name on the upper left corner, starting with your family name then followed by
your given name. (ex. Millora, Michelle)
 Date of activity will be placed across your name.
 The subject plus the schedule will be written just below the name. (ex. PHSENG2 8:00 –
9:30 TTH)
 The type of activity should be placed just below the date. (ex. Q#1, A#2, S#3)

 NOTE: All activities, including assignments, are to be


handwritten. Printed activites will not be accepted.

e. The rubrics to be used for problem solving are as follows:

PROBLEM SOLVING RUBRIC

Points If…

There is evidence of analytic evaluation of the problem with


5 complete details.

The student clearly understands how to solve the problem.

A Self-regulated Learning Module 4


Main concepts are used for the analytic evaluation of the problem
with some lacking details.
4
The student understands the problem-solving techniques but
resulted in a different answer.

Some concepts are used for the analytic evaluation of the


problem and most details are lacking.
3
The student partially understood the problem but is not able to
complete the solution.

Main concepts are not presented for the evaluation of the


problem.
f. 2 Attendance
The student has a poor understanding of the problem but
attempted to solve the problem.

Different concepts not related to the problem are presented.


1 The student did not understand the problem with illustrations or
solution not related to the problem.

0 The student wrote nothing or almost nothing.

4. Learning Outcomes

At the end of this course, the student should be able to:

a. Solve problems in force statics and kinematics.


b. Apply Newton’s Laws of Motion.
c. Solve work and energy problems and apply the law of conservation of energy.
d. Solve problems on impulse, momentum and collisions.
e. Determine stress and strain on a body.
f. Describe characteristics of fluids at rest and in motion and solve problems on these.
g. Describe three methods of heat transfer and solve basic problems.
h. Discuss wave properties, modes of vibration and solve shm problems.
i. Define basic electrical units and measurements.
j. Discuss basic concepts on electricity and apply electrical laws and theorems in a
given circuit.
k. Describe electromagnetism and apply its principles on magnetic field and torque.
l. Describe image formation by mirrors and lenses and solve basic optics problems.

A Self-regulated Learning Module 5


5. Grading system

The requirements for each grading period shall be a minimum of 70% to obtain a passing grade.

The highest percentage rating that may be given in any grading period is 99 and the lowest
passing grade is 75. Any grade lower than 75 is considered “Failed”.

Towards the computation of student’s grades in any subject, the following are done:
1. The student’s actual total score, which refers to the sum of the student’s scores in
recitation, problem set, seat works, quizzes, and grading examination, is determined (RS
= raw score).
2. The student’s actual score is transmuted using the distribution table based on the highest
possible score.(note: the instructor will show the distribution table to the students).

There are three (3) grading periods (first grading, midterms & finals) for each semester. The
cumulative system of computing grades shall be followed. Grades computed for midterms and
finals are considered tentative. The final midterm grade is calculated by getting 1/3 of the first
grading grade plus 2/3 of the tentative midterm grade and the final grade is computed by getting
1/3 of the midterm grade plus 2/3 of the tentative final grade.

6. Study Guide
Going through the modules can be both fun and meaningful learning experience. And the
key to passing this subject is in your hands. Hereunder are some guidelines in making use of
your time and resources efficiently:

a. Take time in reading and understanding each lesson. It is better to be slow


but has surely understood the lesson than to hurry finishing each module and missing
out on the concepts you are supposed to learn. Remember, this subject is a pre-
requisite to your major subjects, so it is very important for you to be able to grasp
each concept and internalize it.

b. Do not jump from one chapter to another. The lessons are usually arranged
such that one lesson is built upon another, thus an understanding of the first is
important in understanding the succeeding lessons.

A Self-regulated Learning Module 6


c. Do not procrastinate. Set a time to do the activities in this subject and stick to it.
Remember that you also have other activities in your other modules. That is why, it
is best to make a study guide and post it where you can easily see it so that
overlapping of activities to be done will be avoided.

d. Be honest when answering the given activities and the test items. Develop the ability
to work independently and be committed to finish the task that you are required
to do. Before answering the activities given to you, please read thoroughly the
instructions and make sure that you submit all the tasks on time. Be self-motivated,
you can do it!

e. Don’t hesitate to ask. If you need to clarify something, feel free to ask me through
whatever ways and means, but in a respectful manner. But, before you do so, make
sure that you have read the module part or instruction thoroughly. There are times
that the matter that you need to clarify is already in the discussions.

B. ATTENDANCE

 3 absences = name will be dropped sa OSA


 In this case, go to the OSA, and get a re-admission slip
 3 lates= 1 absence
 For Asynchronous: reaction to instructions in our class gc will serve as your
attendance
 For f2f – an affidavit of undertaking is required
 Uniform: M- red jacket or formal attire with red polo and slacks
Th- black jacket
F- organizational shirt

A Self-regulated Learning Module 7


*In this module, we will study Mechanics which is a branch of physical science that deals with energy
and forces and their effect on bodies and their application to everyday life.
 The following are the lessons we will be taking up in this module:

I. Measurements and conversion of units


II. Vectors
III. Motion

Learning Outcomes:

 At the end of this module, the student should be able to:


1. Explain the concepts on Kinematics.
2. Discuss competently the concepts of measurements, vectors and motion parameters.
3. Apply these concepts by solving examples using step by step methods.
4. Solve problems correctly and confidently.

KINEMATICS is a branch of Mechanics that focuses on the motion of objects.


Motion is a fundamental observable phenomenon. The study of kinematics involves describing,
measuring and analyzing motion without considering the forces and masses involved in that
motion. Uniformly accelerated motion is described in terms of relationships between measurable
scalar and vector quantities, including displacement, speed, velocity, acceleration and time.
Representations, such as, graphs and vectors, and equations of motion, can be used
qualitatively and quantitatively to describe and predict linear motion.

LESSON I: Measurements and conversion of units


 How can the motion of an object be described using more than just words?
 Measurements of physical quantities which are expressed in terms of standard units, can
do the trick here. For example, the length of a race, which is a physical quantity, can be
expressed in meters (for sprinters) or kilometers (for long distance runners). Without
standardized units, it would be extremely difficult for scientists and engineers to express
and compare measured values in a meaningful way.

 There are two major systems of units used in the world: SI units (acronym for the
French Le Système International d’Unités, also known as the metric system), and

A Self-regulated Learning Module 8


English units (also known as the imperial system). English units were historically used in
nations once ruled by the British Empire. Today, the United States is the only country
that still uses English units extensively. Virtually every other country in the world now
uses the metric system, which is the standard system agreed upon by scientists and
mathematicians.

 The three basic units of measurement are:

1. Length (L) – is the extent of an object from end to end


-The meter (m), or metre, it is defined as the distance traveled by light in a vacuum
over a time interval of 1/299,792,458 of a second.

2. Mass (m) – is the amount of matter in an object


-The kilogram (kg) is defined as the mass of a platinum-iridium kilogram sample
maintained by the Bureau International des Poids et Mesures in Sevres, France.

3. Time (t) – is the measured period during which an action take place
-The second (s) was originally based on a “standard day” of 24 hours, with each
hour divided in 60 minutes and each minute divided in 60 seconds. However, we
now know that a complete rotation of the Earth actually takes 23 hours, 56 minutes,
and 4.1 seconds.

 Conversion of units:

 To measure quantities, units of measurements are required. There are times when the
units used for the measurement, do not match the measurement preference and
convenience as well as the standards prescribed for certain processes and
applications. Converting such units to an extent that it can be understood directly and
applied properly is important.

 Several steps can be followed in the conversion of units:


a. Write the conversion as a fraction
b. Multiply with the conversion factor. Make sure that if the units to be
converted are in the numerator, the conversion factor should be in the
denominator, so that the unwanted units can be cancelled.
c. Cancel same units that are in the numerator and denominator.

 Examples:
1. Change the speed of 20 cm/s to km/hr
𝑐𝑚 1𝑚 1𝑘𝑚 3600𝑠
 20 𝑠 x 100𝑐𝑚 x 1000𝑚 x 1ℎ𝑟 = 0.72 km/hr

2. Convert 15 kg/m3 to g/cm3


𝑘𝑔 1000𝑔 1𝑚
 15 𝑚3 x 1𝑘𝑔 x (100𝑐𝑚 )3 = (15x1000)/1000000 = 0.015 g/cm3

A Self-regulated Learning Module 9


Activity 1.1
PROBLEM SOLVING: Copy the questions in your formatted short bond papers and show your
complete and neat solutions. No erasures allowed.

1. Imagine that you are driving your car in London. As you're driving along, you notice that the
speed limit signs have numbers like 100 (on the highway) and 30 (in the city). As you start to
speed up, you realize that the signs are in mi/hr. Unfortunately, your speedometer only reads in
km/hr. How fast should you be going in km/hr if the speed limits are: a) 100 mi/hr, and b) 30
mi/hr?
2. You are buying carpet to cover a room that measures 45 ft by 30 ft. The carpet cost 200
pesos per square yard. How much will the carpet cost?
3. An average human heart beats 60 times per minute. If the average person lives to the age of
80 years, how many times does the average heart beat in a lifetime?

LESSON II: Vectors


 Physical quantities can usually be placed into two categories, vectors and scalars.

Scalars require only the magnitude. Scalars can be thought of as numbers, they do not
have a direction. They are used primarily to represent physical quantities for which a
direction does not make sense. Some examples of these are: mass, height, length,
volume, and area. Talking about the direction of these quantities has no meaning and so
they cannot be expressed as vectors.

Vectors require both a magnitude and a direction. The vector is represented by an


arrow. The arrow has two parts that define it. The two parts are its length which
represents the magnitude and its direction with respect to some set of coordinate axes.
The greater the magnitude, the longer the arrow. Physical concepts such as
displacement, velocity, and acceleration are all examples of quantities that can be
represented by vectors. Each of these quantities has both a magnitude (how far or how
fast) and a direction. In order to specify a direction, there must be something to which
the direction is relative. Typically this reference point is a set of coordinate axes like the
x-y plane, as shown by the graph below.

A Self-regulated Learning Module 10


 Methods of Vector Addition
1. Graphical or head-to-tail method

- The graphical method of vector addition is also known as the head-to-tail method. To start,
draw a set of coordinate axes. Next, draw out the first vector with its tail (base) at the origin of
the coordinate axes. For vector addition it does not matter which vector you draw first since
addition is commutative, but for subtraction ensure that the vector you draw first is the one you
are subtracting from. The next step is to take the next vector and draw it such that its tail starts
at the previous vector’s head (the arrow side). Continue to place each vector at the head of the
preceding one until all the vectors you wish to add are joined together. Finally, draw a straight
line from the origin to the head of the final vector in the chain. This new line is the vector result
of adding those vectors together.

- in using this method, the following materials are needed: ruler, protractor, pencil

 Example:
A woman takes a walk in the park nearby. Starting from her home, she headed 10 meters
toward South, then turned southeast and walked 20 meters and finally walked another 10
meters east. Find the resultant vector representing her displacement, using graphical method.
Given: D1 = 10 m South
D2 = 20 m SE
D3 = 10 m E
Find: R
Solution:
- since the units are in meters, and definitely we cannot plot the distances on a
bondpaper, hence we need to use a scale.

Scale: 1m:1cm
Thus, in plotting, we use D1 = 10 cm South
D2 = 20 cm SE (angle is exactly 45 from east and also
45 from south, or just midpoint of south and east)
D3 = 10 cm E

A Self-regulated Learning Module 11


a. Draw the cartesian plane with the x and y coordinates

Home(origin)
, direction
D1

Resultant, R(from origin to destination)


45

D2

D3 Destination
b. After drawing D1, D2 is drawn at the head of D1, then D3 is drawn from the head of D2 in
the direction stated.
c. The resultant, R, is drawn from the origin of the first vector, D1, to the head of the last
vector, D3, as shown in the drawing above.
- We will verify the actual values, using the trigonometric method.

Watch the videolink below for more discussions on this topic.


https://www.youtube.com/watch?v=eh6s53xflpY

2. Trigonometric/Component method

Before applying this method, let us first have a brief review of the basic trigonometric
functions that we are going to use in the analysis of this method. Refer to the figure
below for the equations:

A Self-regulated Learning Module 12


To get the component of a vector, use the trigonometric functions, as shown below,

 Example:
A woman takes a walk in the park nearby. Starting from her home, she headed 10 meters
toward South, then turned southeast and walked 20 meters and finally walked another 10
meters east. Find the resultant vector representing her displacement, using component
method.
Given: D1 = 10 m South
D2 = 20 m SE
D3 = 10 m E

A Self-regulated Learning Module 13


Find: R
Solution:
a. Plot all of the vectors in only 1 cartesian plane

D3

45

D1 D2

b. Determine the component of each vector, it is best to use a table to present the values,

Vector X-component,m Y-component,m


D1 0 -10
D2 20 cos 45 =14.1421 -20 sin 45 =- 14.1421
D3 10 0
X,Y 24.1421 -24.1421

c. After solving the sum of X and Y components, use Pythagorean theorem to solve for the
resultant vector.
R = √𝑋2 + 𝑌 2 = √24.14212 + (−24.1421)2 = 34.1421 m
𝑌 −24,1421
The direction,  = tan−1 𝑋 = tan−1 24.1421 = - 45

Thus, the resultant vector, R = 34.1421 m, 45 south of east or simply SE

Watch the videolink below to have a bette understanding of this concept:


https://www.youtube.com/watch?v=j8xt6A0Sd70

Activity 1.2

PROBLEM SOLVING: Copy the questions in your formatted short bond papers and show
your complete and neat solutions. No erasures allowed.

1 . Starting from the center of the town, a car travels West for 80 km and then turns 60 north
of east for another 192 km at which point it runs out of gas. Find the resultant of the car from
the center of the town, using both graphical and trigonometric methods. (15 points for each
method)

2. Solve the resultant of the following vector displacements: 300 cm at 0, 400 cm at 60 and
400 cm at 150. (diagram-5 points, solution-10 points)

A Self-regulated Learning Module 14


LESSON III: Motion
Kinematics is the description of motion without considering its causes. In this module, it
is limited to motion along a straight line, called one-dimensional motion. We are going
to consider both horizontal and vertical motions, as well the combination of these two.

A. Horizontal Motion

Since Kinematics describes motion, it is important to know the important terms in which
this motion can be described, and they are as follows:
1. Distance (d or x)
– refers to the total length traveled by an object
- it is a scalar quantity
2. Displacement (d or x) , same as the resultant
– refers to the straight line distance from the origin of motion to its destination, similar
to the resultant vector we have discussed previously, thus it is a vector quantity.
- it also refers to the change in position of an object
- Position refers to the location of an object at a particular time.
3. speed (V)
– measures how fast an object is moving without regard to its direction.
- average speed is equal to the ratio of total length traveled over the total time of travel.
- instantaneous speed is the speed at a specific point in time
- basic unit: m/s; V=d/t
4. velocity (V)
- it is equal to the displacement divided by the time over which displacement occurs
- instantaneous velocity refers to the velocity at a specific instant or time point
- basic unit: m/s; V=d/t
5. acceleration (a)
- it refers to the rate of change in velocity or the change in velocity over time
- instantaneous acceleration refers to the acceleration at a specific point in time
- basic unit: m/s2; a=V/t= (Vf-Vi)/t
The following basic motion equations will be utilized for our discussions on
kinematics:
Where:
Vf = final speed/velocity
Vi= initial speed/velocity
Vav= average speed/velocity
t = time

A Self-regulated Learning Module 15


 Examples:
1. A biker travels at a rate of 25 km/hr for 5 minutes, then at 50 km/hr for 10 minutes and finally
at 20 km/hr for another 20 minutes before coming to a stop. Find the total distance traveled and
the average speed for the complete trip.

1ℎ𝑟
Given: v1 = 25 km/hr, t1= 5 min x = 0.0833 hr
60𝑚𝑖𝑛
1ℎ𝑟
v2 = 50 km/hr, t2= 10 min x 60𝑚𝑖𝑛
= 0.1667 hr
1ℎ𝑟
v3 = 20 km/hr, t3= 20 min x 60𝑚𝑖𝑛
= 0.3333 hr
Solution:
a) total distance (ST)= S1+S2+S3
𝑘𝑚
x1 = v1t1 = 25 ℎ𝑟 x 0.0833 hr = 2.0825 km
𝑘𝑚
x2 = v2t2 = 50 x 0.1667 hr = 8.335 km
ℎ𝑟
𝑘𝑚
x3 = v3t3 = 20 ℎ𝑟
x 0.3333 hr = 6.666 km
Thus, xT = 2.0825+8.335+6.666 = 17.0835 km

𝑆𝑇 17.0835
b) Vave = total distance/total time = 𝑡𝑇
= 0.0833+0.1667+0.3333
= 29.2877 km/hr

2. An eagle moving at 160 km/hr slowed uniformly to a stop in a time of 1 minute. Find its
acceleration in m/s2 and the distance traveled before stopping.
1ℎ𝑟
Given: Vi = 160 km/hr, Vf= 0 (stopped), t= 1 minx60𝑚𝑖𝑛 = 0.0167 hr

Solution:
160𝑘𝑚
𝑉𝑓−𝑉𝑖 0− 𝑘𝑚 1000𝑚 1ℎ𝑟
a) a = 𝑡
= ℎ𝑟
0.0167ℎ𝑟
= -9600 ℎ𝑟2
x 1𝑘𝑚
x (3600𝑠)2 = -0.7407 m/s2
 What does a negative acceleration mean??? deceleration

𝑉𝑓 2−𝑉𝑖 2 02 −(160𝑘𝑚/ℎ𝑟)2
b) from eq. 5, x = = = 1.3333 km
2𝑎 2(−9600𝑘𝑚/ℎ𝑟2 )

x=Vave(t) = (Vf+Vi)/2 x t = (0+160)/2 x 0.0167 = 1.336 km


1
x= Vit + 2 at2 = 160(0.0167) + ½ (-9600)(0.0167)2 = 1.3333 km

ACTIVITY 1.3

PROBLEM SOLVING: Copy the questions in your formatted short bond papers and show your
complete and neat solutions. No erasures allowed.

1. An car’s velocity decreases uniformly from 20 m/s to 6 m/s while covering 70 m


distance in a straight line. Find its acceleration and the time taken to travel the
distance.

A Self-regulated Learning Module 16


2. A runner is accelerating uniformly as it passes two checkpoints that are 30 m apart. The time
taken to reach checkpoint two, from checkpoint one, is 4.0 s, and the runner’s speed at the first
checkpoint is 5.0 m/s. Find the runner’s acceleration and its speed at the second checkpoint.

B. Vertical Motion

Free fall is the motion of a body where its weight is the only force acting on an
object.
The acceleration of free-falling objects is called the acceleration due to gravity, since
objects are pulled towards the center of the earth.
The acceleration due to gravity is constant on the surface of the Earth and has the
value of 9.80 m/s2.

- The best way to see the basic features of motion involving gravity is to start by considering
straight up and down motion with no air resistance or friction. This means that if the object is
dropped, the initial velocity is zero. Once the object is in motion, the object is in free-fall. Under
these circumstances, the motion is one-dimensional and has constant acceleration, g.

The kinematic equations for objects experiencing free fall are:

Where:

V= final velocity, same as Vf

Vo= initial velocity, same as Vi

Y or h = vertical displacement

 For motion going down, g is +, (+g); while for motion going up, g is -, (-g).

 Examples:
1. A mango falls freely from its branch. Find (a) the distance it falls after 3 sec, and (b) the time
required to reach a speed of 25 m/s.

Given: Vi= 0
Solution:
1 1
a) using y = Vot + 2 gt2 = 0(3) + 2 (9.8)(3)2 = 44.1 m
b) from equation (a), we can derive a formula for time, where gt = V-Vo
𝑉−𝑉𝑜 25−0
thus, t = 𝑔 = 9.8 = 2.551 sec

A Self-regulated Learning Module 17


2. A baseball is thrown straight upward with a speed of 30 m/s. (a) How long will it reach the
highest point? (b) How high will it rise? (c) How long after it leaves the hand will it return to the
starting point? Vfup=0, V1down=0

Given: Vo = 30 m/s=Vi Viup, Vfdown= 30 m/s


Solution:
a) when the baseball reaches the highest point, it will stop momentarily before it starts
to travel downwards back to its origin, thus, the final velocity for the upward motion is
𝑉−𝑉𝑜 0−30
zero. Using equation (a), we have; t = = = 3.0612 sec, since the motion is
𝑔 −9.8
upward, then g is negative(-).

b) Using equation (d) to solve for Y, we have,


2
𝑉2 −𝑉𝑜2 0−(30)
Y= 2𝑔
= (2)(−9.8)
= 45.9184 m
c) the total time for the object to return to its original position is twice the time to go up
or twice the time to go down, thus
total time = 2xt = 2x(3.0612) = 6.1224 sec

ACTIVITY 1.4

PROBLEM SOLVING: Copy the questions in your formatted short bond papers and show your
complete and neat solutions. No erasures allowed.

1. A bottle dropped from a balloon reaches the ground in 20 s. Determine the height of the
balloon if (a) it was at rest in the air and (b) it was ascending with a speed of 50 m/s when the
bottle was dropped.

2. A vase is thrown straight downward with initial speed 8.0 m/s from a height of 25 m. Find (a)
the time it takes to reach the ground and (b) the speed with which it strikes.

C. Projectile Motion

Projectile motion is the motion of an object thrown or projected into the air, subject to only the
(vertical) acceleration due to gravity. The object is called a projectile, and its path is called its
trajectory.
We analyze two-dimensional projectile motion by breaking it into two independent one-
dimensional motions along the vertical and horizontal axes. The horizontal motion is
simple, because the horizontal acceleration is zer0, ax=0 and thus making the horizontal
velocity, Vx a constant value. The velocity in the vertical direction begins to decrease as
an object rises; at its highest point, the vertical velocity is zero. As an object falls
towards the Earth again, the vertical velocity increases again in magnitude but points in
the opposite direction to the initial vertical velocity. The x and y motions can be
recombined to give the total velocity at any given point on the trajectory.

A Self-regulated Learning Module 18


Let us consider the 2 cases of projectile motion:

1. Horizontal projection

Where: u = initial velocity, Vi

R= range, which is the maximum horizontal


distance traveled, equal to

R = Vit

- since the velocity is horizontal, then Vi=Vix and


Viy=0(has no vertical component)

2. Projected at an angle

Where: Vo = initial velocity

Voy = Vo sin

Vx = Vo cos

 Examples:
1. As shown in the figure, a projectile is fired horizontally with a speed of 30 m/s from the top
of a cliff that is 80 m high. (a) How long will it take to strike the level ground at the base of the
cliff? (b) How far from the foot of the cliff will it strike? Range,x

Vix =30 m/s, Viy = 0

Vi=Vix=30 m/s

Y= Vy

V=d/t=x/t

A Self-regulated Learning Module 19


Solution:

a) The horizontal and vertical motions are independent of each other. Consider first the
vertical motion,
1 1 1
y= Viyt + 2
gt2 = 0(t) + 2
(9.8)t2 = 2 (9.8)t2 = 80

2𝑦 2(80)
thus, t2 = 9.8 and t = √ 9.8
= 4.0406 s

b) Now consider the horizontal motion. For it, a=0 since the horizontal motion is
constant, thus Vix=Vfx = 30 m/s. Then, using the value of t found in (a), we have

x = Vixt = 30 (4.0406) = 121.2183 m

2. A baseball is thrown with an initial velocity of 100 m/s at an angle of 30.0° above the
horizontal, as shown in figure below. How far from the throwing point will the baseball attain its
original level?

Vi = 100 m/s
Vfy(up)=0

Y(max height) = (Vfy2-Viy2)/2g =(0-502)/2(-9.8)= 127.55 m


Solution:
Solve for Vix = Vicos30= 100cos30 = 86.6025 m/s
Solve for Viy = Vi sin30= 100sin30 = 50 m/s

From, X = Vixt, we need to solve for the total time first,


Taking into consideration the vertical motion, we can use the formula
𝑉𝑓𝑦−𝑉𝑖𝑦 𝑉𝑓𝑦−𝑉𝑖𝑦 0−50
g= 𝑡 , thus, t = 𝑔
= −9.8 = 5.102 sec, this time is only for the motion
going up, to solve for the total time, ttotal = 2xt = 2x5.102 = 10.2041 sec

Thus, X = 86.6025m/s(10.2041s) = 883.7005 m

ACTIVITY 1.5

PROBLEM SOLVING: Copy the questions in your formatted short bond papers and show your
complete and neat solutions. No erasures allowed.

A Self-regulated Learning Module 20


1. A marble, rolling with speed 20 cm/s, rolls off the edge of a table that is 80 cm high. (a)
How long does it take to drop to the floor? (b) How far, horizontally, from the table edge
does the marble strike the floor?

2. A body is projected downward at an angle of 30° with the horizontal from the top of a
building 170 m high. A Its initial speed is 40 m/s. (a) How long will it take before striking
the ground? (b) How far from the foot of the building will it strike? (c) At what angle with
the horizontal will it strike?

SUMMARY

Let us now summarize the topics that we have learned from this module:

1. Kinematics is the study of motion without regard to mass or force


2. Scalars are physical quantities represented by a single number and no direction.
3. Vectors are physical quantities that require both magnitude and direction.
4. The motion quantities include distance, displacement, speed, velocity and acceleration.
5. Free fall is the motion of a body where its weight is the only force acting on an object.
6. The acceleration of free-falling objects is called the acceleration due to gravity, g.
7. Projectile motion is the motion of an object that is subject only to the acceleration of gravity.
8. We analyze two-dimensional projectile motion by breaking it into two independent one-
dimensional motions along the vertical and horizontal axes.

ASSESSMENT (Quiz)

A Self-regulated Learning Module 21


• In this module, we will we discuss how the direction and magnitude of motion is affected by the
factors that surround a moving object.

 The following are the lessons we will be taking up in this module:

I. Factors affecting the motion of an object.


II. Newton’s Laws of Motion

Learning Outcomes:

 At the end of this module, the student should be able to:

1. State the three Laws of Motion.


2. Apply Newton’s Laws of Motion in problem solving.

Dynamics considers the forces that affect the motion of moving objects and systems. Newton’s
laws of motion are the foundation of dynamics. These laws provide an example of the breadth
and simplicity of principles under which nature functions. They are also universal laws in that
they apply to similar situations on Earth as well as in space.

Lesson I. Factors that affect the motion of an object include:


1. Force
- defined generally as an agency of change
- it is that quantity that changes the velocity of an object
- its unit is the newton (N), or dyne, or pound (lb); where 1 N = 0.225 lb

Classifications of Forces:

a. External force (F) – one whose source lies outside of the body being considered, also
referred to as the applied force.

b. Weight (W) – is the gravitational force acting downward on the object of mass m.
- it is equal to the product of mass and gravitational acceleration
- W = mg, in newton

c. Tension (T) – force that acts on strings, chains or tendons, acting to stretch these or we
can say that it is the pulling force that acts along a stretched flexible connector, such as a
rope or cable. When a rope supports the weight of an object that is at rest, the tension in
the rope is equal to the weight of the object: T = W = mg.

A Self-regulated Learning Module 22


This Figure shows; (a) Tendons in the finger carry
force T from the muscles to other parts of the
finger, usually changing the force’s direction, but
not its magnitude (the tendons are relatively
friction free). (b) The brake cable on a bicycle
carries the tension T from the handlebars to the
brake mechanism. Again, the direction but not the
magnitude of T is changed.

d. Friction (f)- a force that opposes the motion of an object and is parallel to the surface
with which the object is in contact.
- Friction has two types, namely, a) kinetic friction which is a force that opposes the
motion of two systems that are in contact and moving relative to one another, and
b) static friction which is a force that opposes the motion of two systems that are in
contact and are not moving relative to one another

e. Normal force (N) – a force that is perpendicular to the surface where the object is in
contact.
- the word “normal” means perpendicular to the surface.

Free-body diagram (FBD)


- are diagrams used to show the relative magnitude and direction of all forces acting upon an
object in a given situation. The figure below is an example of a free body diagram showing all
the forces acting on a body.

Fx=max; F-f = max


Fy=may; N-W= may

surface

A Self-regulated Learning Module 23


2. Mass (m)

- is the amount of quantity of matter in an object, measured in kilograms (kg)


- it is also a measure of the amount of inertia in an object

 Inertia refers to the property of a body to remain at rest or to remain in motion with
constant velocity.

Lesson II. Newton’s Laws

There are three laws of motion formulated by Sir Isaac Newton, namely;

1. Newton’s first law of motion:

- states that, “a body at rest remains at rest, or, if in motion, remains in motion
at a constant velocity unless acted on by a net external force”

- also known as the Law of Inertia


- Newton’s first law is completely general and can be applied to anything from an object
sliding on a table to a satellite in orbit to blood pumped from the heart. Experiments
have thoroughly verified that any change in velocity (speed or direction) must be caused
by an external force.

2. Newton’s second law of motion: Law of Acceleration

- We know that a change in motion is equivalent to a change in velocity. A


change in velocity means, by definition, that there is an acceleration. Newton’s first law
says that a net external force causes a change in motion; thus, we see that a net
external force causes acceleration.

- the second law states that, “The acceleration of a system is directly


proportional to and in the same direction as the net external force acting on the system,
and inversely proportional to its mass”.

In equation form, Newton’s second law of motion is,


𝑭
a= ; where: a is in m/s2
𝒎
F is in newton(N) or kg.m/s2
m is in kg

 Examples:
1. A 5.0 kg object is to be given an upward acceleration of 0.30 m/s2 by a rope pulling straight
upward on it, as shown in the figure. What must be the tension in the rope?

A Self-regulated Learning Module 24


Given:

T Figure (b) shows the free body diagram for the given problem.

The weight, W=mg = 5(9.8) = 49 N

Using Newton’s 2nd Law, we have Fy=may,


Where, Fy is the sum of all forces along the y-axis, taking
consideration of the sign in which upward motion is (+) and
downward motion is (-), ay is the acceleration along the y-axis
W

Thus, Fy=T-W=5kg(0.30 m/s2) =1.5N, and


T = 1.5N +W = 1.5N + 49N = 50.5N

2. A horizontal force of 140 N is needed to pull a 60.0 kg box across the horizontal floor at
constant speed as shown in the figure. What is the coefficient of friction between floor and box?

coefficient of friction () – is the ratio between the force necessary to move one surface
𝑓 𝑓𝑟𝑖𝑐𝑡𝑖𝑜𝑛
horizontally over another. In equation: = = (unitless)
𝑁 𝑛𝑜𝑟𝑚𝑎𝑙

The figure shows the free body diagram.


At constant speed, the acceleration is zero, ax=0.

Thus, Fx=max=0, so that,


F-f = 0, and, 140-f=0 or f=140N
And, Fy=may, ay=0, since the object is horizontally
moving, hence there is no vertical acceleration, thus,
N-W = 0 or N=W=mg=60(9.8) = 588N
𝑓 140
Therefore,  = 𝑁
= 588
= 0.2381

ACTIVITY 2

PROBLEM SOLVING: Copy the questions in your formatted short bond papers and show your
complete and neat solutions. No erasures allowed.

1. The weight of an astronaut plus his space suit on the Moon is only 200 N. How much does he
weigh on Earth? What is his mass on the Moon? On Earth?
2. A 30-kg crate hangs at the end of a long rope. Find its acceleration when the tension in the
rope is a)250N, and b) 150 N
3. A 12 kg- box is released from the top of an incline that is 5 m long and makes an angle of
50 to the horizontal. A 60-N friction force slows down the motion of the box. A) What is the
acceleration of the box and, b) how long will it take to reach the bottom of the incline?

A Self-regulated Learning Module 25


3. Newton’s third law: Law of action and reaction

- states that, “ Whenever one body exerts a force on a second body, the first
body experiences a force that is equal in magnitude and opposite in direction to the
force that it exerts.
 This law represents a certain symmetry in nature: Forces always occur in pairs, and one body
cannot exert a force on another without experiencing a force itself. We sometimes refer to
this law loosely as “action-reaction,” where the force exerted is the action and the force
experienced as a consequence is the reaction.

SUMMARY

Let us now summarize the topics that we have learned from this module:
1. Inertia is the tendency of an object to remain at rest or remain in motion. Inertia is related to
an object’s mass.

2. Mass is the quantity of matter in a substance.

3. An external force is one acting on a system from outside the system, as opposed to internal
forces, which act between components within the system

4. Forces are given many names, such as push, pull, thrust, lift, weight, friction, and tension.
They have been grouped into several categories and given names relating to their source, how
they are transmitted, or their effects.

5. The three laws that govern the motion of an object, called Newton’s Laws of motion, state in
the first law that, “an object will not change its motion unless a force acts on it”. In the second
law, “the force on an object is equal to its mass times its acceleration”. In the third law, “when
two objects interact, they apply forces to each other of equal magnitude and opposite
direction”.

ASSESSMENT (Quiz #2)

A Self-regulated Learning Module 26


In this module, we will we discuss situations where the object moves in a curve. We begin the
study of uniform circular motion by defining the angular quantities needed to describe rotational
motion.

 The following are the lessons we will be taking up in this module:

A. Angular Displacement
B. Angular velocity
C. Angular Acceleration

Learning Outcomes:
 At the end of this module, the student should be able to:

1. Define angular quantities.


2. Solve angular quantities.

When objects rotate about some axis, they follow a circular path. Here are some quantities
used to define angular motion.

A. Angular displacement ()


- is defined as “the angle in radians (degrees, revolutions) through which a point or line
has been rotated about a specified axis”. It is the angle of the movement of a body in a
circular path.

- it is equal to the ratio of the arc length(s) and the radius(r), as shown in the figure.
𝒔
=𝒓;

Where:  - angular displacement in radian(rad)


r – radius in meter (m)
s – arc length in meter (m)

Conversion factor: 2rad = 360 degrees = 1 complete rotation or revolution

B. Angular velocity ()


- is the rate at which an object rotates, or revolves, about an axis, or at which the
angular displacement between two bodies changes. In equation,
𝒇−𝒊
= ; where:  - angular speed in rad/s, rev/s(RPS), rev/min(RPM)
𝒕
f – final angular displacement
i – initial angular displacement

A Self-regulated Learning Module 27


t – time in seconds

C. Angular acceleration ()


The angular acceleration is the time rate of change of the angular velocity and
is designated by  and expressed in radians per second squared. In equation,

𝒇−𝒊
= ; where:  - angular acceleration in rad/s2
𝒕
f – final angular velocity
i – initial angular velocity
t – time in seconds

EQUATIONS FOR UNIFORMLY ACCELERATED ANGULAR MOTION are exactly analogous


to those for uniformly accelerated linear motion as seen on the table below.

Examples:

1. A wheel of 40-cm radius rotates on a stationary axle. It is uniformly speeded up from


rest to a speed of 900 rpm in a time of 20 s. Find (a) the constant angular acceleration of
the wheel and (b) the tangential acceleration of a point on its rim.

Solution:

A Self-regulated Learning Module 28


A pulley of 5.0-cm radius, on a motor, is turning at 30 rev/s and slows down uniformly to 20
rev/s in 2.0 s. Calculate (a) the angular acceleration of the motor, (b) the number of
revolutions it makes in this time, and (c) the length of belt it winds in this time.

Solution:

ACTIVITY 3

PROBLEM SOLVING: Copy the questions in your formatted short bond papers and show your
complete and neat solutions. No erasures allowed.

1. A car has wheels of radius 30 cm. It starts from rest and accelerates uniformly to a speed of
15 m/s in a time of 10.0 s. Find the angular acceleration of its wheels and the number of rotations
one wheel makes in this time.

2. Express 80.0 deg/s in (a) rev/s, (b) rev/min, and (c) rad/s

3. Through how many radians does a point on the Earth’s surface move in 6.00 h as a result of
the Earth’s rotation?

4. A car wheel 25 cm in radius is turning at a rate of 8.0 rev/s when the car begins to slow
down uniformly to rest in a time of 14 s. Find the number of revolutions made by the wheel and
the distance the car goes in 14 s.

SUMMARY

Let us now summarize the topics that we have learned from this module:

1. Uniform circular motion is motion in a circle at constant speed.


2. Arc length IS the distance traveled by an object along a circular path.
3. Radian is a unit of angle measurement.
4. The quantities used to describe angular motion are angular displacement, angular velocity
and angular acceleration.

Assessment (Quiz#3)

A Self-regulated Learning Module 29


 In this module, we will we discuss about the very basic concept when dealing with forces
which is the idea of equilibrium or balance.

 The following are the lessons we will be taking up in this module:

A. First Condition
B. Second Condition

Learning Outcomes:

 At the end of this module, the student should be able to:

1. Discuss competently the concepts on translational and rotational motion, and torque.
2. Apply vectors in solving equilibrium problems.

When all the forces that act upon an object are balanced, then the object is said to be in a state
of equilibrium. Balance of forces mean that the algebraic sum of all the components of the
forces along the x and the y components must be ZERO. From the figure below, we can say
that the X = 0 ,since it is -30N+30N, and the Y = 0, since it is equal to 50N-50N.

Thus, Equilibrium is the state of a body at rest or in uniform motion, the resultant of all forces
on which is zero.

A. First Condition of Equilibrium-Translational Equilibrium

-For an object to be in equilibrium, it must be experiencing no acceleration. This means that


both the net force and the net torque on the object must be zero. Here we will discuss the first
condition, that of zero net force.

- In the form of an equation, this first condition is: Fnet=0.

- this condition requires that, the algebraic sum forces acting along each axis of motion
must be equal to zero, and happens where the object’s velocity is zero or where the object is
moving at a constant velocity.

A Self-regulated Learning Module 30


For two-dimensional motion, the net external forces along the typical x and y axes are zero.
This is written as,

Fx = 0; Fy = 0

 Examples:

1. A 50-N box is slid straight across the floor at constant speed by a force of 25 N, as shown in
figure. a) How much friction force impedes the motion of the box? (b) How large is the normal
force? (c) Find coefficient of kinetic friction(µk) between the box and the floor.

A Self-regulated Learning Module 31


2. The block in the figure shown below is at rest. The angle of the incline is slowly increased. At
an angle of 42°, the block begins to slide. What is the coefficient of static friction(s) between
the block and the incline?

FBD:

Wx f

Wy

Solution:

Using the FBD, solve for the N, Fy=0, we have,

To find the frictional force, f, Fx=0, we have,

B. Second Condition of Equilibrium-Rotational Equilibrium

-The second condition of static equilibrium says that the net torque acting on the object must
be zero.

 = 0

A Self-regulated Learning Module 32


- we can say that the sum of the clockwise torques around an axis is equal to the sum of the
counterclockwise torques around the same axis.

- A rotating body or system can be in equilibrium if its rate of rotation is constant and remains
unchanged by the forces acting on it.

- The magnitude of torque about the axis of rotation is given by the formula: =rFsinθ.

Where:  = torque, defined as a rotational or twisting effect of a force; Torque is a measure of


the force that can cause an object to rotate about an axis; its SI unit is newton-
meter(Nm), or the unit foot-pound (ft-lb)

r = is the moment arm or the distance of the object from the axis of rotation
F = force in newton
 = angle between the force and the distance, r

This figure illustrates how torque is experienced by


a rotating object due to a force applied to it from
the axis.

axis

 Examples:
1. Find the torque about axis A due to each of the forces shown.

=rFsin

F2

F1, 1=rF1sin= 0.8m(20N)sin0=0N

F3

A Self-regulated Learning Module 33


2. Where must a 0.80-kN object be hung on a uniform, horizontal, rigid 100-N pole so that a girl
pushing up at one end supports one-third as much as a woman pushing up at the other end?

Given:

A Self-regulated Learning Module 34


ACTIVITY 4

PROBLEM SOLVING: Copy the questions in your formatted short bond papers and show your
complete and neat solutions. No erasures allowed.

1. In the figure shown below, the system is in equilibrium. (a) What is the maximum value
that the hanging weight can have if the friction force on the 40-N block cannot exceed
12.0 N? (b) What is the coefficient of static friction between the block and the tabletop?

N T2y

T2

f=12N T1 T1 T2x=T2cos30

Fx=0, T1-f=0; T1=f=12N Fw

Fy=0, N-W=0; N=W=40N

=f/N= 12/40=0.3 W=40N Fx=0, T2x-T1=0; T2x=T1=12N

T2=T2x/cos30 = 12/cos30=13.86N

Fy=T2y-Fw=0; T2y=Fw=T2sin30

Fw=6.93N

2. Two people, at the ends of a uniform beam that weighs 400 N, hold the beam vertically.
How much vertical force must each person furnish to the beam for it to be in
equilibrium? Draw the free body diagram.

SUMMARY

Let us now summarize the topics that we have learned from this module:

1. “Equilibrium” means equal balance or it is a state of balance.


2. An object is in translational equilibrium when the vector sum of all the forces acting on it is
zero.
3. Rotational equilibrium says that the net torque acting on the object must be zero.
4. Torque is the ability of an object to rotate about a given axis.

A Self-regulated Learning Module 35


 In this module, we will we discuss about mechanical work, energy and power which gives us
more tools to use to approach physical situations.

 The following are the lessons we will be taking up in this module:

A. Work
B. Energy
C. Power

Learning Outcomes:
 At the end of this module, the student should be able to:

1. Discuss competently the concepts on mechanical work, energy and power.


2. Solve work and energy problems and apply the Law of conservation of energy.

In physics, we say that work is done on an object when you transfer energy to that object. If
one object transfers energy to a second object, then the first object does work on the second
object. How fast this transfer of energy happens is given by power.

A. WORK
- The work done on a system by a constant force is the product of the component of the force
in the direction of motion times the distance through which the force acts. This is expressed in
equation form as:
Wk = Fd cos θ, where: Wk is work,
F is the magnitude of the force on the system,
d is the magnitude of the displacement of the system, and
θ is the angle between the force vector F and the
displacement vector d.

- The SI units, for work and energy are measured in newton-meters (n-m) or simply joule.

1J = 1N · m = 1 kg · m2/s2.

- One joule is not a large amount of energy; it would lift a small 100-gram apple a
distance of about 1 meter.

-Work is the transfer of energy by a force acting on an object as it is displaced.

A Self-regulated Learning Module 36


-The work done by a force is zero if the displacement is either zero or perpendicular to the
force.

-The work done is positive if the force and displacement have the same direction, and negative
if they have opposite direction.

B. ENERGY
- is the ability to do work
- Work and energy have the same units
- the two types of energy are:

a. Kinetic energy (KE) - the energy an object has by reason of its motion, equal to
1 1
KE = 2
mv2 = 2 m(Vf2 – Vi2)

Where: m=mass in kg
Vf=final speed in m/s
Vi-iniyial speed in m/s

b. Gravitational potential energy (PEg) - the energy an object has by reason of its
position, equal to,

PEg = mgh= Wh

Where: m=mass in kg
h=height in m
g=gravitational acceleration in m/s2

The work-energy theorem states that the net work, Wnet on a system changes its kinetic
energy, meaning, when there is a net force doing work on an object, the object's kinetic energy
will change by an amount equal to the work done, thus

𝟏
o Wnet= KE: F.d = m(Vf2 – Vi2)
𝟐

The law of conservation of energy states that energy can not be created or destroyed, it
can merely be changed from one form of energy to another. Energy often ends up as heat,
which is thermal energy of atoms and molecules.

C. POWER
- Power is the rate at which work is done, or energy is transferred per unit time, in equation
form,
𝑊𝑜𝑟𝑘
P= 𝑡𝑖𝑚𝑒 , J/s

- The SI unit for power is the watt (W), where 1 W=1J/s


- The power of many devices such as electric motors is also often expressed in horsepower
(hp), where 1 hp = 746 W.

A Self-regulated Learning Module 37


- It is a scalar quantity.

 Examples:
1. The object, in the figure, is being pulled along the ground by a 75-N force directed
28° above the horizontal. How much work does the force do in pulling the object
thru a distance of 8.0 m?

2. A 0.50-kg block slides across a tabletop with an initial velocity of 20 cm/s and comes to rest
in a distance of 70 cm. Find the average friction force that retarded its motion.

3. An advertisement claims that a certain 1200-kg car can accelerate from rest to a speed of 25
m/s in a time of 8.0 s. What average power must the motor produce to cause this acceleration?
Ignore friction losses.

A Self-regulated Learning Module 38


ACTIVITY 5

PROBLEM SOLVING: Copy the questions in your formatted short bond papers and show
your complete and neat solutions. No erasures allowed.

1. What is the power output, in watts and in hp, for a 50.0-kg woman who runs up a
3.00 m high flight of stairs in 3.50 s, starting from rest but having a final speed of
2.00 m/s? (hint: The work going into mechanical energy is W= KE + PE. At the bottom
1
of the stairs, we take both KE and PEg as initially zero; thus, W=KEf+PEg=2 mvf2+mgh)

2. a) How fast must a 3000-kg elephant move to have the same kinetic energy as a 60.0-kg
sprinter running at 10.0 m/s? (b) Discuss how the larger energies needed for the movement
of larger animals would relate to metabolic rates.

3. A 200-kg cart full of Physics books is pushed slowly up an incline. How much work does the
pushing force do in moving the cart up to a platform that is 2 m above the starting point if
friction is negligible?

SUMMARY

Let us now summarize the topics that we have learned from this module:

1. Work is the application of a force over a distance.


2. Energy can be defined as the capacity for doing work.
3. Kinetic energy is the energy in motion while potential energy is the energy at rest.
4. Energy can neither be created nor destroyed, but only transformed from one kind to another.
5. Power is simply the time rate of doing work.

A Self-regulated Learning Module 39


 In this module, we will we discuss about the relationship of mass, force and time in the
motion of objects.

 The following are the lessons we will be taking up in this module:

A. Momentum
B. Impulse
C. Collision

Learning Outcomes:

 At the end of this module, the student should be able to:

1. Solve problems on impulse, momentum and collisions.


2.Apply the law of conservation of energy to problems.

If you want to change the momentum of an object, you have to apply a force. But if you
apply that force for a long time then the momentum will change more. and the effect of
the force and the time the force is applied, is the impulse of the force.

A. Momentum
- is a quantity that describes an object's resistance to stopping (a kind of "moving
inertia").
- is represented by the symbol p
- is the product of an object's mass and velocity, in equation:
p = mv = m(Vf-Vi)

- is a vector quantity, and has the unit, kg.m/s

B. Impulse
- is a quantity that describes the effect of a net force acting on an object (a kind
of "moving force").
- is represented by the symbol J.
- is the product of the average net force acting on an object and its duration, in
equation,
J = F∆t

- is a vector quantity, and has the unit, N-s or (kg.m/s2).s = kg.m/s

A Self-regulated Learning Module 40


Impulse-Momentum Theorem

- The impulse-momentum theorem states that the change in momentum of an


object equals the impulse applied to it, hence
J = ∆p

F∆t = m∆v

- The impulse-momentum theorem is logically equivalent to Newton's second


law of motion.
- These units of impulse and momentum are equivalent [N-s = kg.m/s]

C. Collision or impact
- Basically refers to the sudden, forceful coming together in direct contact of two bodies,
such as in the case of two billiard balls, a golf club and a ball, a hammer and a nail
head, two railroad cars when being coupled together, or a falling object and a floor.
-
When the only forces on the colliding bodies are those exerted by the bodies
themselves, the principle of conservation of momentum states that the total momentum
of the system is unchanged in the collision process, thus when two masses of m1 and
m2 collide,

Total momentum before impact = total momentum after impact


m1Vi1 + m2Vi2 = m1Vf1 + m2Vf2

 where Vi1 and Vi2 are the velocities before impact, and Vf1 and Vf2 are the
velocities after impact

- Based on whether kinetic energy is conserved or not conserved, collisions are classified
as elastic or inelastic, respectively.
 In an elastic collision, mechanical energy is conserved; that is, the total kinetic
energy of the system of particles after collision equals the total kinetic energy before
collision, in equation,
KEbefore = KEafter

 For inelastic collisions, however, the total kinetic energy after collision is different
from the initial total kinetic energy.

- The coefficient of restitution(e), indicates how elastic or inelastic the collision is. A
coefficient of restitution equal to zero indicates a perfectly inelastic collision, in which
the colliding bodies stick together after the collision. For a perfectly elastic collision,
e=1, while for inelastic collisions, e < 1. If the bodies stick together after collision, e=0.

 Examples:
1. A 2.0-kg brick is moving at a speed of 6.0 m/s. How big a force is needed to stop the
brick in a time of 7.0 × 10-4 s?

A Self-regulated Learning Module 41


2. A 16-g marble is moving toward the +x-direction at 30 cm/s while a 4.0-g marble is
moving toward the -x-direction at 50 cm/s. They collide head on and stick together.
Find their velocity after the collision.

ACTIVITY 6

PROBLEM SOLVING: Copy the questions in your formatted short bond papers and show your
complete and neat solutions. No erasures allowed.

1. A 7.00-g bullet moving horizontally at 200 m/s strikes and passes through a 150-g tin
can sitting on a post. Just after impact, the can has a horizontal speed of 180 cm/s.
What was the bullet’s speed after leaving the can?
2. Two balls of equal mass, moving with speeds of 3 m/s, collide head-on. Find the
speed of each after impact if they stick together.

SUMMARY

Let us now summarize the topics that we have learned from this module:

1. Momentum(p) is the product of mass and its velocity.


2. Impulse(J) is the product of force and the time over which it acts on an object.
3. An impulse causes a change in the momentum of an object.
4. When collision occurs, the momentum or kinetic energy is transferred from one object to
another. Thus, in collisions and explosions, the vector sum of the momenta just before the
event equals the vector sum of the momenta just after the event.
5. The coefficient of restitution(e), shows how elastic or inelastic the collision is.
Assessment (Quiz #4)

A Self-regulated Learning Module 42


 In this module, we will we discuss about the effect of force on the stress and strain
experienced by a body in which the force acts.

Learning Outcomes:

 At the end of this module, the student should be able to:

1. Determine the stress and strain on a body.


2. Solve problems on stress and strain.

To a greater or lesser extent, most solid materials exhibit elastic behavior, but there is a
limit to the magnitude of the force and the accompanying deformation within which
elastic recovery is possible for any given material.

ELASTICITY
— Property of a body to return to its original size and shape when the forces that deformed it are
removed, such as on plastics, foams, rubbers, metals, etc.

 STRESS ( σ )
— is a force or combination of forces distributed throughout the whole of an object that acts to
deform it.

— In equation:
σ = F/A ,
where: F denotes the force acting on a body, in newton; and
A denotes the area over which the force acts, in m2

— Units of stress are Newtons per squared meter (N/m 2), or the same as units of pressure - Pascal
(symbol: Pa), in honor of the French mathematician Blaise Pascal (1623–1662)

*Gigapascal (GPa) is more appropriate unit in which 1 GPa = 1x109 Pa

 STRAIN (ε )
— it is a measure of how much an object is stretched or deformed
— Strains take the general form of a change in one geometric quantity divided by the original
value of that quantity or a similar quantity with the same unit (∆ℓ/ℓ0, ∆V/V0, etc.).
— Strains are always dimensionless or unitless ratios [m/m, m 3/m3, etc.]

 Modulus of Elasticity
1. Stress is directly proportional to strain.
2. An elastic modulus is the ratio of some stress to the strain (σ / ε ).
3. The SI unit of all elastic moduli is the pascal or newton per meter squared [Pa = N/m2],
but gigapascals [1 GPa = 109 Pa] are more commonly used.
4. Elastic moduli are properties of materials, not the objects made from those materials.

A Self-regulated Learning Module 43


 Modulus: Three types

1. Young's modulus (Y) is the ratio of tensile stress to tensile strain or compressive stress to
compressive strain. In equation:
𝒔𝒕𝒓𝒆𝒔𝒔 𝝈
Y = 𝒔𝒕𝒓𝒂𝒊𝒏 = 𝜺

 Tension and compression


1. stress
 Tensile stress is the normal force per area (σ = F/A) that causes an
object to increase in length.
 Compressive stress is the normal force per area (σ = F/A) that causes an
object to decrease in length.

2. strain
 Tensile strain is the fractional increase in length of an object (ε = ∆ℓ/ℓ0)
due to a tensile stress.
 Compressive strain is the fractional decrease in length of an object
(ε = ∆ℓ/ℓ0) due to a compressive stress.

 Example:
A metal wire 75.0 cm long and 0.130 cm in diameter stretches 0.0350 cm when a load
of 8.00 kg is hung on its end. Find the stress, the strain, and the Young’s modulus for the
material of the wire

Solution:

A Self-regulated Learning Module 44


2. Shear Modulus (S)
- The shear modulus or rigidity modulus is the ratio of shear stress to shear strain.

 stress
1. Shear stress is the tangential force per area (σ = F/A) that causes one
face of an object to become displaced parallel to the opposite face.
2. Shear stress changes rectangles into parallelograms.
 strain
1. Shear strain is the fractional tangential displacement relative to the
normal distance between opposite parallel faces (ε = ∆x/y) caused by a
shear stress.
2. Shear strain is the tangent of the shear angle.

 Example:

A box-shaped piece of gelatin dessert has a top area of 15 cm 2 and a height of 3.0 cm.
When a shearing force of 0.50 N is applied to the upper surface, the upper surface displaces 4.0
mm relative to the bottom surface. What are the shearing stress, the shearing strain, and the
shear modulus for the gelatin?

Solution:

A Self-regulated Learning Module 45


3. Bulk Modulus (B)
- The bulk modulus or compression modulus is the ratio of the increase in pressure to the
relative decrease in volume.

 stress
1. Pressure is the compressive stress (σ =P = F/A) applied uniformly to all surfaces
of an object.
2. Uniform compression or decompression changes the volume of objects but not
their shape.

 strain
1. Volume strain is the fractional change in volume of an object (ε = ∆V/V0) due to
a change in pressure.

 The strength of a material is a measure of its ability to withstand a load without breaking.

 EXAMPLE:
The bulk modulus of water is 2.1 GPa. Compute the volume contraction of 100 mL of
water when subjected to a pressure of 1.5 MPa.

Solution:

A Self-regulated Learning Module 46


ACTIVITY 7

PROBLEM SOLVING: Copy the questions in your formatted short bond papers and show your
complete and neat solutions. No erasures allowed.

1. A 200-kg load is hung on a wire of length 4 m, cross-sectional area 2 cm2, and Young’s modulus
of 8 x 1010 N/m2. What is it’s increase in length?
2. For safety in climbing, a mountaineer uses a nylon rope that is 50 m long and 1 cm in diameter.
When supporting a 90-kg climber, the rope elongates 1.6 m. Find Young’s modulus.
3. Compute for the volume change of a solid copper cube, 50 mm on each edge, when subjected
to a pressure of 20 MPa. The bulk modulus for copper is 125 GPa.

SUMMARY
Let us now summarize the topics that we have learned from this module:
1. Elasticity is a measure of the deformation of an object when a force is applied. Objects that are
very elastic like rubber have high elasticity and stretch easily.
2. Stress is force over area.
3. Strain is change in length over original length.

A Self-regulated Learning Module 47


 In this module, we will we discuss about the fundamentals of fluid mechanics in a very
general manner so that you can understand the way that forces are produced and
transmitted by fluids that are, at rest and in motion, and to apply the physical principles
behind some of the most common applications of fluid mechanics in the conservation of
energy and Newton's laws of motion.

Learning Outcomes:

 At the end of this module, the student should be able to:

1. Solve basic problems in fluid statics and kinematics.


2. Describe the characteristics of fluids at rest and in motion.

FLUIDS
— are substances that continually deforms (flows) under an applied shear stress, or external force
— Fluid mechanics is essentially the study of fluids either in motion or at rest.

 PRESSURE (P)
 is the ratio of normal force to area.
 Although both force and area are vectors, pressure is a scalar quantity and has no
direction.
 Pressure is a way to describe force in a region of a continuous system and has the
equation:
𝑭
P=𝑨
 Units:
— The SI unit of pressure is the pascal [Pa = N/m2 = kg/ms2].
— One hundred thousand pascals are sometimes called
a bar [100,000 Pa = 1 bar].
— Standard Atmospheric Pressure(atm) = 1.013 x 105 Pa

Hydrostatic Pressure
— is the pressure that is exerted by a fluid at equilibrium at a given point within
the fluid, due to the force of gravity. It increases in proportion to depth
measured from the surface because of the increasing weight of fluid exerting
downward force from above.
— is due to a column of fluid of height(h) and density(), and has the equation:
P = gh, where g is the gravitational acceleration

A Self-regulated Learning Module 48


The figure on the left, shows the relationship of pressure to
the depth of the fluid.

Pressure in a fluid (P) at rest


— is equal to the weight of a column of fluid divided by the area on which it rests,
so that
P = P0 + ρgh

— increases uniformly with depth (h)


— is directly proportional to the density of the fluid (ρ)
— depends on the surface pressure (P0)
— acts equally in all directions and therefore exerts a net force
perpendicular to any surface that it contacts

Pascal's principle (Blaise Pascal): “When the pressure on any part of a confined fluid
(liquid or gas) is changed, the pressure on every other part of the fluid is also changed
by the same amount.”

BUOYANCY (also known as the buoyant force)


— is the force exerted on an object that is wholly or partly immersed in a fluid.
— The symbol for the magnitude of buoyancy is B or FB
— As a vector it must be stated with both magnitude and direction. Buoyancy acts
upward for the kind of situations encountered in everyday experience.
— As with other forces, the SI unit of buoyancy is the newton [N].
— Buoyancy is caused by differences in pressure acting on opposite sides of an
object immersed in a static fluid.

Typical situations:
 The pressure on the bottom of an object is greater than the top
(since pressure increases with depth).
 The force on the bottom pushes up and the force on the top pushes
down
(since force is normal to the surface).
 The direction of the net force due to the fluid is upward.

A Self-regulated Learning Module 49


Archimedes' Principle: “The magnitude of the buoyant force on an object is equal to the
weight of the fluid it displaces”.
 B = ρgVdisplaced

densities B > Wobject B = Wobject B < Wobject

object rises float on surface


ρobject < ρfluid
(wholly immersed ) (partly immersed)

neutral buoyancy
ρobject = ρfluid
(wholly immersed)

ρobject > ρfluid object sinks

Relationship of Buoyancy and density

VISCOSITY (also known as dynamic viscosity, absolute viscosity, or simple viscosity)


— Is represented by the Greek letter η (eta).
— defined informally as the quantity that describes a fluid's resistance to flow.

 Factors Affecting Viscosity


a. Viscosity varies with material. (Viscosity is a property of materials.)
b. The viscosity of simple liquids decreases with increasing temperature and increases
under very high pressures.
c. The viscosity of gases increases with increasing temperature is independent of
pressure and density.

Fluid flow or discharge(J) : When a fluid that fills a pipe flows through the pipe
with an average speed v, the flow or discharge, J, is
J = Av
— where A is the cross-sectional area of the pipe
— The units of J are m3/s in the SI
— Sometimes J is called the rate of flow or the discharge rate.

 EXAMPLES:
1. A partly filled beaker of water sits on a scale, and its weight is 2.30 N. When a piece of metal
suspended from a thread is totally immersed in the beaker (but not touching bottom), the scale
reads 2.75 N. What is the volume of the metal?

A Self-regulated Learning Module 50


2. Find the pressure due to the fluid at a depth of 76 cm in still (a) water (pw = 1.00 g/cm3) and
(b) mercury (p = 13.6 g/cm3).

3. Oil flows through a pipe 8.0 cm in diameter, at an average speed of 4.0 m/s. What is the flow J
in m3/s and m3/h?

ACTIVITY 8

Using the formatted paper, copy and answer (handwritten) completely the given questions
below.
1. State as many as you can the applications of buoyancy and viscosity in:
a) your everyday life.
b) your chosen field of engineering.
2. A certain town receives its water directly from a water tower. If the top of the water in the
tower is 26.0 m above the water faucet in a house, what should be the water pressure at
the faucet?

A Self-regulated Learning Module 51


 In this module, we will we discuss Heat transfer as a discipline of thermal engineering that
concerns the generation, use, conversion, and exchange of thermal energy (heat) between
physical systems.

Learning Outcomes:

 At the end of this module, the student should be able to:

1. Solve basic problems in heat transfer.


2. Describe the three methods of heat transfer.

HEAT (Q) is the transfer of internal energy from one region to another, from a system at a
higher temperature to a system at a lower temperature.

Units: Joule (J) or calorie (cal), where 1 cal = 4.184 J


British thermal unit (BTU) = 1054 J

 THREE WAYS TO TRANSFER HEAT ENERGY:

1. CONDUCTION
 Heat conduction (as opposed to electrical conduction) is the flow of internal energy from a region
of higher temperature to one of lower temperature by the interaction of the adjacent particles
(atoms, molecules, ions, electrons, etc.) in the intervening space.

Note: it's the rate (P or Φ) at which heat is transferred, not the amount (Q) of heat transferred.

∆Q dQ
Where: P= ∆t
= dt

 Factors affecting the rate of heat transfer by conduction.


1. temperature difference, ∆T, in °C
2. length, ℓ, in m
3. cross-sectional area, A in m2
4. thermal conductivity of material, k

2. CONVECTION
 is the transfer of heat by the flow of a fluid.
 Spontaneous convection is can be summarized in two simple rules
 hot fluid rises
 cold fluid sinks
 will result in the formation of closed loops of circulating fluid called convection
cells
 Forced convection is aided by fans, blowers, impellers, lung power, etc.

A Self-regulated Learning Module 52


3. RADIATION
 Heat radiation (as opposed to particle radiation) is the transfer of internal energy in the
form of electromagnetic waves. For most bodies on the Earth, this radiation lies in the
infrared region of the electromagnetic spectrum.

 EXAMPLES:
1. An iron plate 2 cm thick has a cross-sectional area of 5000 cm2. One face is at 150 °C, and
the other is at 140 °C. How much heat passes through the plate each second? For
iron, kT = 80 W/m.K.

2. A beverage cooler is in the shape of a cube, 42 cm on each inside edge. Its 3.0-cm thick
walls are made of plastic kT 0.050 W/m.K . When the outside temperature is 20 °C,
howmuch ice will melt inside the cooler each hour?

ACTIVITY 9
Using the formatted paper, copy and answer (handwritten) completely the given questions
below.

1. Rub the palm of your hand on a metal surface for 30 to 45 seconds. Place the palm of your other
hand on the unrubbed portion of the surface and then on the rubbed portion. The rubbed portion will
feel warmer. Now, repeat this process on a wooden surface. Why does the temperature difference
between the rubbed and unrubbed portions of the wood surface seem larger than for the metal
surface?

2. A tile floor may feel uncomfortably cold to your bare feet, but a carpeted floor in an adjoining room
at the same temperature feels warm. Why?

3. On a very hot day, it’s possible to cook an egg on the hood of a car. Would you select a black car or
a white car on which to cook the egg? Why?

Assessment (Quiz #5)

A Self-regulated Learning Module 53


 In this module, we will we study oscillatory motion and waves, the underlying principles that
describe all of them and the type of force that underlies the simplest oscillations and waves.

Learning Outcomes:

 At the end of this module, the student should be able to:

1. Solve simple harmonic motion applications.


2. Discuss the properties of waves, modes of vibration of strings and air columns.

WAVE MOTION
— is a means of transferring energy from one place to another without the transfer of
matter. A wave on the surface of a body of water, for example, can transfer energy
over long distances while the particles within the medium, the water, move only by
jiggling locally as the overall disturbance, the wave, passes. The media through
which waves propagate may be liquids, solids, gases, or even a vacuum in the case
of electromagnetic waves.

A. MECHANICAL WAVES
- is a disturbance in matter that transfers energy through a medium

 Classifying waves by medium:


*medium is where the wave travels.

1. Mechanical waves
— Matter is the medium. ( air, liquid and solid)
— Sound is a mechanical wave.

2. Electromagnetic waves
— Electric and magnetic fields are the media.
— Light is an electromagnetic wave.
— Electromagnetic waves include, radio waves, microwaves, infrared, light, ultraviolet, x-
rays, and gamma rays.

3. Gravitational waves
— The gravitational field is the medium.

 Classifying mechanical waves by type of disturbance:

1. Transverse waves
— The disturbance is perpendicular to the direction of propagation, usually in strings,
cables and ropes.
— All electromagnetic waves are transverse. This includes light.
— A crest is a point of maximal change in the positive direction.

A Self-regulated Learning Module 54


— A trough is a point of maximal change in the negative direction.
— The diagrams below show how a transverse wave is produced.

The two parts of a transverse wave are:


1. Crest- highest point in the wave
2. Trough-lowest point in the wave.

2. Longitudinal or compressional waves

- The disturbance is parallel to the direction of propagation, usually in columns


- Sound is a longitudinal wave.
- A compression or condensation is a region where the medium is under compression, in
which the particles are near each other.
- A rarefaction or dilation is a region where the medium is under tension, in which the
particles are far from each other.
- The two parts of longitudinal waves are: a) compressed part where the particles are
near each other, and b) stretched part where the particles are far from each other as
shown in the diagram below.

A Self-regulated Learning Module 55


B. PERIODIC WAVES
— A periodic wave is one that repeats as a function of both time and position and can be
described by its amplitude, frequency, wavelength, speed, and energy.

 Characteristics of periodic waves:

1. Amplitude (A)
- The maximum displacement of a wave.

2. Period (T)
- The time between successive cycles of a repeating sequence waves
- The ratio of time(t) to the number of cycles (n), where
𝑡𝑖𝑚𝑒 𝑡
T= =
𝑐𝑦𝑐𝑙𝑒𝑠 𝑛
- The SI unit of period is the second [s].

A Self-regulated Learning Module 56


3. Frequency (f )
- The number of cycles of a repeating sequence of events in a unit interval of time
- The ratio of the number of cycles(n) to time(t), where
𝑐𝑦𝑐𝑙𝑒𝑠 𝑛
f = 𝑡𝑖𝑚𝑒 = 𝑡
- Frequency and period are reciprocals (or inverses) of one another.
1 1
f = 𝑇; T=𝑓

- The SI unit of frequency is the hertz or cycles per second(cps)

4. Wavelength (λ, the Greek letter lambda)


- The distance between any point on a periodic wave and the next nearest point
corresponding to the same portion of the wave
- Wavelength is measured between adjacent points in a wave.
- The SI unit of wavelength is the meter [m].

5. Speed (v)
- Waves propagate with a finite speed (sometimes called the wave speed) that depends
upon,
a. the type of wave,
b. the composition of the medium, and
c. the state of the medium.

- Is the product of frequency and wavelength for periodic waves, where


v = f λ, m/s or
𝑻𝒆
v=√𝒎 ;
𝑳
where, Te is the tension in the string or cable, in newton
m/L is the mass per unit length of the string or cable, in kg/m

A Self-regulated Learning Module 57


- Frequency and wavelength are inversely proportional.
 Lower frequency waves have longer wavelengths.
 Higher frequency waves have shorter wavelengths.

 Examples:
1. A uniform flexible cable is 20 m long and has a mass of 5.0 kg. It hangs vertically under
its own weight and is vibrated from its upper end with a frequency of 7.0 Hz. (a) Find the
speed of a transverse wave on the cable at its midpoint. (b) What are the frequency and
wavelength at the midpoint?
Solution:

2. A 60 cm long bar, clamped at its middle, is vibrated lengthwise by an alternating force at


its end. (See figure) Its fundamental resonance frequency is found to be 3.0 kHz. What is
the speed of longitudinal waves in the bar?

Solution:

A Self-regulated Learning Module 58


C. PERIODIC MOTION
- Oscillatory motion is repetitive and fluctuates between two locations.
- Periodic motion repeats a cycle of motion with a characteristic time.
- A period is the time to complete one cycle of periodic motion.
- Hooke’s Law states that “The net force on the system is directly proportional to the
displacement of the system from its equilibrium position.” In equation:

∑F = −kx, where:
k is the spring constant, because it is a quantity that varies
between springs, and has the SI unit newton per meter [N/m]
x is the displacement
F is a restoring force (acts in a direction opposite the displacement)

 Simple harmonic motion (abbreviated SHM)


- is a motion that obeys Hooke’s Law.
- is both oscillatory and periodic.

 The two objects that undergo SHM are:


1. Simple pendulum
2. Spring-mass system

D. SOUND
- is a mechanical, longitudinal wave.
- As a mechanical wave, sound requires a medium.
- Sound cannot propagate through a vacuum.
- There is no sound in outer space.
- Sound is produced by small and rapid pressure changes.
- Vibrating objects produce periodic sound waves.
 Sources include human vocal cords, musical instruments (strings,
brass, reeds, drums, bells), speakers, buzzers, motors, etc.
- Implosive or explosive pressure changes produce sound pulses.
 Sources include plosive consonants (p, t, k), finger snapping,
crunchy foods, electrical sparks, thunder, whip cracking, gunshots,
etc.
- The speed of sound depends upon the medium and its state.
 Sound usually travels fast in gases, faster in liquids, and fastest in
solids.
 The speed of sound in air increases with temperature.
 The speed of sound in air at 0 °C is 331 m/s, and changes by 0.61
m/s for every 1°C change in temperature
 The speed of sound in air at 20 °C is 343 m/s

 The amplitude of a sound wave corresponds to its intensity or loudness.


 The intensity of a sound is a measure of its power density usually measured on a
logarithmic scale
 The loudness of a sound is its intensity as perceived by the human ear.

A Self-regulated Learning Module 59


 The frequency of a sound wave corresponds to its pitch.
 The upper frequency limit for human hearing is around 18,000 to 20,000 Hz.
 Frequencies above the range of human hearing are ultrasonic.
 The lower frequency limit for human hearing is around 18 to 20 Hz.
 Frequencies below the range of human hearing are infrasonic.

 A reflected sound wave is known as an echo.

 The Doppler effect


- is the apparent change in the frequency of a wave caused by relative motion between
the source of the wave and the observer.
- applies to all types of waves.
- increases the frequency of a wave (raises the pitch of a sound wave) when the source
moves toward the observer.
- decreases the frequency of a wave (lowers the pitch of a sound wave) when the source
moves away from the observer.

 EXAMPLES:
1. When a 400-g mass is hung at the end of a vertical spring, the spring stetches 35 cm.
What is the spring constant of the spring, and how much further will it stretch if an
additional 400-g mass is hung from it?

2. An explosion occurs at a distance of 6.00 km from a person. How long after the explosion
does the person hear it? Assume the temperature is 14.0 °C.
Solution:

A Self-regulated Learning Module 60


ACTIVITY 10

Using the formatted paper, copy and solve the given problems below. Show your complete and very
neat solution. Write legibly and clearly. Round off final answers to two decimal places and enclose
your final answers in a box.

1. Transverse waves with a speed of 50.0 m/s are to be produced on a stretched string. A
5.00-m length of string with a total mass of 0.060 kg is used. (a) What is the required
tension in the string? (b) Calculate the wave speed in the string if the tension is 8.00 N.

2. An object attached to a spring vibrates with simple harmonic motion as described by


Figure P13.42. For this motion, find (a) the amplitude, (b) the period, (c) the angular
frequency, (d) the maximum speed, (e) the maximum acceleration, and (f) an equation for
its position x in terms of a sine function.

3. An object–spring system undergoes simple harmonic motion with an amplitude A. Does the
total energy change if the mass is doubled but the amplitude isn’t changed? Are the kinetic
and potential energies at a given point in its motion affected by the change in mass?
Explain.

4. (a) You are driving down the highway in your car when a police car sounding its siren
overtakes you and passes you. If its frequency at rest is f 0, is the frequency you hear
while the car is catching up to you higher or lower than f 0? Explain your answer. (b) What
about the frequency you hear after the car has passed you? Explain your answer.

ASSESSMENT (QUIZ #6)

A Self-regulated Learning Module 61


 In this module, we will we study the basic principles of electrostatics and explain how objects
become charged and describe the effect of those charges on other objects in contact.

Learning Outcomes:

 At the end of this module, the student should be able to:

1. Compute the electric force between electric charges;


2. Define electric field, electric potential and capacitance

A. ELECTROSTATICS

 The classical study of electricity is generally divided into three general areas.
1. Electrostatics: the study of the forces acting between charges, called electricity at
rest.
2. Electric current: the study of the forms of energy associated with the flow of charge.
3. Electromagnetism: the study of the forces acting between charges in motion.

 Charge is the fundamental quantity of electricity. (Electricity is all about charge.)


- No one can tell you what charge is. They can only tell you how charges interact.
 The types of charges are:
a. proton - positive (+)
b. electron - negative (−)

- A neutron is a subatomic particle found in the nucleus of every atom. The particle
derives its name from the fact that it has no electrical charge; it is neutral.

- The term neutral does not refer to a third type of charge, but to the presence in a
region of positive and negative charges in equal amount.
 The sum of identical positive and negative quantities is zero (0). This is what it
means to be electrically neutral.
 Meaning, a neutral body has the same number of electron and protons in it.

A Self-regulated Learning Module 62


 Types of charged bodies are:
1. Positively charged – there are more protons than electrons in the body.
2. Negatively charged – there are more electrons than protons in the body.
3. A neutral body becomes either positively or negatively charged whenever an atom
losses or gains an electron

 Methods of charging
1. Conduction- is the transfer by contact with an already charged object
2. Induction- is the separation of charge when near an already charged object with no
physical contact
- Charged objects can attract electrically neutral objects through induction.
Here's how:
• Like charges move away from the charged object.
• Opposite charges move toward the charged object.
• Attraction predominates since the like charges are closer together than
the opposite charges.

 Classification of materials:
1. Conductors- materials in which charge moves easily within them, like metals,
electrolytes (ionized liquids), plasmas (ionized gases)
2. Insulators- materials in which charge does not move easily within them, like nonmetals
(pure water, organics, gases)

 The SI unit of charge is the coulomb [C]


- 1 coulomb is the amount of charge transferred by one ampère of current in one second
of time [A. s].
- Elementary charge value is 1.60 × 10−19 coulomb – the magnitude of the charge on
an electron or proton

 LAWS ON ELECTROSTATICS
1. Like charges repel, opposite charges attract.
2. Coulomb's law by Charles Agustin de Coulomb, a French physicist
- States that “The force between two point charges is directly proportional to the
magnitude of each charge (q1, q2), inversely proportional to square of the separation
between their centers (r), and directed along the line connecting their centers (r).”

In equation :

q1q2
F=k
r2
where
k = 9 × 109 Nm2/C2
- is the electrostatic constant

A Self-regulated Learning Module 63


B. ELECTRIC FIELD – imaginary region in space surrounding a charged object

- Electric field lines, as shown by the arrows, comes out of positive charges and goes into
negative charges as shown in the figure below.
- The diagrams show the electric field around a spherically symmetric charge diverges from
positive charge and converges on negative charge
- Field lines never intersect.
- Electric field strength is proportional to line density.

C. ELECTRIC POTENTIAL or voltage or potential difference or electromotive


force(emf)
- The electric potential is the same as voltage (V)
- The electric potential difference between two locations is the work(W) required to
move a charge(q) from one location to another divided by the magnitude of the
charge.
W
∆V =
q
Or V = KQ/r, where: k = 9 x 109;
Q= charge in coulomb;
r= distance between the charge and the point

 Relation of electric potential to electric field


- The electric field at a point is space is the gradient of the electric potential at that point.
calculus versionnon-calculus version
∆V
E = −∇V E=−
d

- The electric potential of a point in space is the path integral of the electric field starting
from a point infinitely far away.

A Self-regulated Learning Module 64


calculus versionnon-calculus version
r

V = − ⌡ E · dr ∆V = −Ed

- The SI unit of electric potential is the volt.


- The electric field has two frequently used, the newton per coulomb and the volt per
meter.

D. EXAMPLES:

1. Three point charges are placed on the x-axis as shown. Find the net force on the -5 µC
charge due to the two other charges.

2. (a) What is the absolute potential at each of the following distances from a charge of 2.0
µC: v = 10 cm and v = 50 cm? (b) How much work is required to carry a 0.05 µC charge
from the point at v = 50 cm to that at v = 10 cm?

Solution:

A Self-regulated Learning Module 65


ACTIVITY 11

Using the formatted paper, copy and solve the given problems below. Show your complete
and very neat solution. Write legibly and clearly. Round off final answers to two decimal places
and enclose your final answers in a box.

1. If the distance between two charges is doubled, by what factor is the magnitude of the electric
force changed?
2. The magnitude of the electric force between two protons is 2.33 N. How far apart are they?

3. A charge of +2 µC charge is placed at the x = 0. Find the absolute potential at a point located
at x=40 cm and at x=100 cm.

ASSESSMENT (QUIZ #7)

A Self-regulated Learning Module 66


 In this module, we will we discuss the basic concepts of electricity, as well as, understand the
basic electrical operation which governs most practical applications of electricity today.

 The following are the lessons we will be taking up in this module:

LESSON 1 - CURRENT, RESISTANCE, VOLTAGE, POWER, ENERGY

LESSON 2 - SERIES, PARALLEL AND COMBINATION CIRCUITS

LESSON 3 - KIRCHHOFF’S LAWS

LEARNING OUTCOMES:

 At the end of this module, the student should be able to:


1. Discuss competently the concepts and laws on electricity.
2. Solve problems on resistance and cells in series and parallel.

LESSON 1: CURRENT, RESISTANCE, VOLTAGE, POWER, ENERGY


 Electricity is the continuous flow of charges through a material.
 Materials could either be conductors or insulators. Conductors allow current to flow thru them
like in case of copper wires, silver, aluminum, etc., while in insulators, current almost could not
flow like in papers, rubber, wood, etc.

A. The Basic Electrical Quantities are as follows:

1. Voltage (V)
 is the difference in electric potential between two points, which is defined as the
work needed per unit of charge to move a test charge between the two points
 also called electric potential or electromotive force (emf)
 unit is in volts

The positive and negative terminals are called polarities, which indicate
the direction of current flow in an electrical circuit.  indicates the emf
which is the rating of the battery, and r is the battery’s internal resistance.

A Self-regulated Learning Module 67


 A voltmeter is a device that measures the voltage or potential between two points.
-This is the symbol for a voltmeter when used in a circuit.

2. Resistance (R)
 is a measure of the opposition to current flow in an electrical circuit
 it is measured in ohms, symbolized by the Greek letter omega (Ω)

A resistor is a passive two-terminal electrical component that implements electrical


resistance as a circuit element. In electronic circuits, resistors are used to reduce current
flow, adjust signal levels, to divide voltages, bias active elements, and terminate
transmission lines, among other uses. The diagram below shows the actual resistors and
their schematic symbols.

 An Ohmmeter is a device that measures the resistance of a material.

3. Current (I)
 is defined as flow of electrons in wire, or the rate of flow of charges thru a wire
 formula for current is I=Q/t, where Q is charge in coulomb (C) & t is time in
second (s)
 its unit is C/s or ampere(A)

 An Ammeter is a device that measures the current through a material in a


circuit.

A Self-regulated Learning Module 68


This is the symbol of an ammeter when used in a circuit.

4. Electrical Power (P)


 is the rate of work done per unit time, at which electrical energy is transferred by
an electric circuit
 it is also the product of current and voltage, P=IV
 its formula is, P=W/t, where W is energy in Joules (J) and t is time in seconds
(s)
 The SI unit of power is the watt or joule per second, and larger unit is the
kilowatt (kw)

5. Electrical Energy
 is the work done by moving electric charges
 Is the product of power and time
 Its formula is W=P*t and its unit is Joule
 Another unit commonly used is the kilowatt hour (kwh)
 We are paying for the electrical energy that we consume which is simply
multiplying the power of the device in kw and the total time the device was used
in hours.

 The meter used to measure the energy that we consume at home is called the KWH
meter.

B. OHM’S LAW (by George Simon Ohm)


 This law, which was formulated by George Simon Ohm, relates the first three
electrical quantities.
 It states that, “Current is directly proportional to the voltage, but inversely to the
resistance.”

 In equation, I=V/R or V=I*R or R=V/I

EXAMPLES:

1. (a) How many electrons will flow thru a 10-W light bulb per second if the current
thru it is 0.75 A? (b) What is the resistance of the bulb? (c) How much voltage is
required to have this current?

Given: I = 0.75 A

P = 10 W

t = 1 sec

A Self-regulated Learning Module 69


Solution:

(a) # of electrons = Q/Qe, where Qe is the magnitude of the charge of an electron


equal to 1.6 x 10-19 C
From I=Q/t, we have Q=It, thus, Q = 0.75A*1s = 0.75 C

# of electrons = 0.75 C / 1.6 x 10-19 C = 4.69 x 1018

(b) R = V/I but there is no voltage given, we can use the power formula in which P
= IV
Using V=IR from Ohm’s Law, substitute this formula in the power formula for V
Thus, P = I * IR = I2*R
Then, R = P/I2 = 10/(0.75)2 = 17.78 

(c) V = IR = 0.75 * 17.78 = 13.33 V

2. Assumimg the cost of energy from the electric company is 8.50 pesos for each
kilowatt hour, what will be the cost per day of operating a lamp that draws a current
of 1.7 A from a 110 V line.

Given: cost of energy =P 8.50/kwh


I = 1.7 A
V = 110 V

Find: cost of energy per day

Solution:

Cost of energy = W in kwh * cost of energy/kwh


W = P*t = VI*t = (1.7x110)*24 hrs/day = 4,488 watt-hr , but 1kw = 1000 watts

Thus, W in kwh = 4,488 watt-hr x 1kw/1000 watts = 4.488 kwh

Cost of energy per day = 4.488 kwh * P8.50/kwh = P38.148 pesos

Activity 12.1 (Quiz)

LESSON 2: SERIES, PARALLEL AND COMBINATION CIRCUITS

 An electrical circuit is a defined as a path for current to flow. Just like a hose which is
used for water to flow through it.
- It is like a pathway made of wires that electrons can flow through. A battery or other
power source gives the force (voltage) that makes the electrons move. When the

A Self-regulated Learning Module 70


electrons get to a device like a light bulb, your computer, or a refrigerator, they give it
the power to make it work.

 There are three basic parts of an electric circuit, namely,


1. Voltage source, like batteries and outlets.
2. Electrical load, like bulbs, appliances, etc.
3. Connecting wires

 The voltage source is like the gate valve and the hose like the connecting wires. Once
the gate valve is opened, water automatically flows through the hose to its destination at
the other end of the hose. The water that flows is like the current in the circuit.
 Also, current only flows when the three important parts of the circuit are present.

 Schematic or electrical diagrams are a graphical representations of an electrical circuit.


A pictorial circuit diagram uses simple images of components, while a schematic diagram shows
the components and interconnections of the circuit using standardized symbolic representations.

Schematic and Pictorial diagrams of a simple circuit

A. SERIES AND PARALLEL CIRCUITS


 Circuits consisting of just one battery and one load resistance are very simple to analyze, but they
are not often found in practical applications. Usually, we find circuits where more than two
components are connected together.

 There are two basic ways in which to connect more than two circuit
components: series and parallel.

1. Series – in this circuit connection the components are connected end-to-end in a line to
form a single path through which current can flow.

A Self-regulated Learning Module 71


Here, we have three resistors (labeled R 1, R 2, and R 3) connected in a long
chain from one terminal of the battery to the other. (It should be noted
that the subscript labeling—those little numbers to the lower-right of the
letter “R”—are unrelated to the resistor values in ohms. They serve only
to identify one resistor from another.)

The defining characteristic of a series circuit is that there is only one path for current to
flow. In this circuit, the current flows in a clockwise direction, from point 1 to point 2 to point 3
to point 4 and back around to 1. Also, the total resistance is equal to the sum of all resistances
connected in series and the total voltage is equal to the sum of the voltage drop across each
resistance.

In equation form:

IT=I1=I2=I3 = constant current

RT=R1+R2+R3

VT=V1+V2+V3

EXAMPLE:
Given the circuit below, determine the total resistance (R T), total current (IT) and
voltage drop for each resistance (V1, V2, V3).

Solution:

a) RT = R1 + R2 + R3 = 3 + 10 + 5 = 18 k or 18,000 
9
b) Using Ohm’s Law, IT = VT/ RT = 18000 = 0.0005 A or 0.5 mA
(milliAmpere) = I1 = I2 = I3
c) V1 = I1R1 = 0.0005*3,000 = 1.5 V
V2 = I2R2 = 0.0005*10,000 = 5.0 V

V3 = I3R3 = 0.0005*5,000 = 2.5 V


 Let us verify if the sum of the voltage drops is equal to the total voltage:

 VT=V1+V2+V3 = 1.5+5.0+2.5 = 9 V

2. Parallel – in this circuit connection, all components are connected across each other’s
leads. In a purely parallel circuit, there are never more than two sets of electrically common
points, no matter how many components are connected. There are many paths for current
flow, but only one voltage across all components.

A Self-regulated Learning Module 72


Electrically common points are shown in this diagram.

Again, we have three resistors, but this time they form more than
one continuous path for current to flow. There’s one path from 1 to 2 to
7 to 8 and back to 1 again. There’s another from 1 to 2 to 3 to 6 to 7 to
8 and back to 1 again. And then there’s a third path from 1 to 2 to 3 to
4 to 5 to 6 to 7 to 8 and back to 1 again. Each individual path (through
R 1, R 2, and R 3) is called a branch .

The defining characteristic of a parallel circuit is that all components are connected
between the same set of electrically common points. Looking at the schematic diagram, we see
that points 1, 2, 3, and 4 are all electrically common. So are points 8, 7, 6, and 5. Note that all
resistors, as well as the battery, are connected between these two sets of points.

In equation form:

VT=V1=V2=V3 = constant voltage

IT=I1+I2+I3

EXAMPLE:

Given the circuit below, determine the total resistance (RT), total current (IT) and current
for each resistance (I1, I2, I3). For parallel circuit, VT=V1=V2=V3, since voltage is
constant.

Vt=V1=V2=V3=9V

Solution:

1 1 1
(a) RT = 1/[( )+( )+( )] = 625 
10,000 2,000 1,000

A Self-regulated Learning Module 73


9
(b) IT = VT/ RT = 625
= 0.0144 A or 14.4 mA
9
(c) I1 = V1/ R1 = 10000
= 0.0009 A
9
I2 = V2/ R2 = = 0.0045 A
2000
9
I3 = V3/ R3 = 1000
= 0.0090 A

 Let us verify if the sum of the current per branch is equal to the total current:

 IT=I1+I2+I3 = 0.0009+0.0045+0.0090 = 0.0144 A

 And, of course, the complexity doesn’t stop at simple series and parallel circuits. We can have
circuits that are a combination of series and parallel, too.

B. Combination Circuits
 These circuits combine the characteristics of both series and parallel.

 Strategy for simplifying series parallel circuits:


a. Begin as far away as possible from the voltage source.
b. Find pair of resistors in the circuit that are in parallel or in series, combine
them using the principles that we have learned previously, and then
replace these series or parallel resistors with their equivalent resistor.
c. Continue, moving toward the voltage source until a single equivalent
resistor represents the entire resistor network.
d. Watch the video on the link provided to learn more about combination
circuits.
https://www.youtube.com/watch?v=-PiB2Xd3P94

 Let us now apply the characteristics of both series and parallel circuits in the given
circuits below to be able to understand further these concepts.

 Given the circuit below, solve for: RT, IT, I1, I2, I3, V1, V2, V3, PT, P1, P2, P3

Solution:
(a) Before solving for the total resistance, examine first the circuit and see if
there are resistances connected in simple series and in simple parallel. In
the circuit, we can see that resistors R1 and R2 are in parallel, thus

A Self-regulated Learning Module 74


1 1 1 1
R12 = 1/( + )= 1/( + )= 81.71
𝑅1 𝑅2 220 130

we can redraw the circuit such that,

R3

o R12



 The original series parallel circuit is now simplified to a series circuit
with two
resistances connected. Combining these resistances will give us a
simple circuit with only one resistor load. Thus, we have the total
resistance as,

RT = R12 +R3 = 81.71+470 = 551.71

𝑉𝑇 12
(b) IT = 𝑅𝑇
= 551.71
= 0.0218A or 21.8 mA

(c) From this circuit, since R12 and R3 are connected in series, then we can say
that same current flows thru them which is the total current, so that
IT=I12=I3=21.8 mA

Knowing the value of I3, we can solve for the voltage drop of R3 using Ohm’s
Law:
V3=I3R3 =0.0218* 470 = 10.246 V

Also, the voltage drop across R12 is, V12=I12R12 = 0.0218*81.71= 1.781 V
Since, R1 and R2 are in parallel, they have the same voltage drop, thus
V1=V2=V12

Therefore, currents for R1 and R2 can now be solved:


𝑉1 1.781
I1 = 𝑅1
= 220
= 0.0081 A or 8.1 mA
𝑉2 1.781
I2 = 𝑅2
= 130
= 0.0137 A or 13.7 mA

A Self-regulated Learning Module 75


(d) The total power taken by the circuit is PT=VTIT= 12*0.0218 = 0.2616 watt
Also the individual power taken by each resistor are:
P1 =V1I1 = 1.781*0.0081 = 0.0144 watt
P2 =V2I2 = 1.781*0.0137= 0.0244 watt
P3 =V3I3 = 10.246*0.0218= 0.2234 watt

 Note that the sum of all the power taken by each resistor is equal to the total power
regardless of their connection whether series, parallel or combination circuits.

 After solving all currents, voltages and powers, it is best to make a table to present your
answers like the one shown below:

Resistance, Current, A Voltage, V Power, watt


R1 = 220 0.0081 1.781 0.0144
R2 = 130 0.0137 1.781 0.0244
R3 = 470 0.0218 10.246 0.2234

Remember this:

The three principles regarding SERIES circuits are:

1. Current: The amount of current is the same through any component in a series circuit.
2. Resistance: The total resistance of any series circuit is equal to the sum of the
individual resistances.
3. Voltage: The supply voltage in a series circuit is equal to the sum of the individual
voltage drops.

The three principles regarding PARALLEL circuits are:

1. Voltage: Voltage is equal across all components in a parallel circuit.


2. Current: The total circuit current is equal to the sum of the individual branch currents.
3. Resistance: Individual resistances diminish to equal a smaller total resistance rather
than add to make the total.

Activity 12.2

PROBLEM SOLVING: Copy the questions in your formatted short bond papers and show
your complete and neat solutions. No erasures allowed.
1. Given the figures below, determine in each figure which resistances are connected in series and
which are connected in parallel, then solve for the total resistance if all resistances are 100
each.

A Self-regulated Learning Module 76


To illustrate, let me do the first figure :
 Examining figure 1, we can say that:
1 1
In parallel – R2 and R3, R23 = 1/(100 + 100)= 50
In series – R23 and R1
Thus, RT = R1+R23 = 100+50= 150

Now, its your turn to do figures 2-5 😊 (5 points each)

2. Given the circuit below, solve completely, systematically and show illustrations of the resistors
being combined, for the following: (a) R T (10 points), (b) IT (5 points), (c) PT (5 points), (d)
voltage and current and power per resistor (12 points)

Make a table to show your final answers for letter (d) (8 points)

LESSON 3: KIRCHHOFF’S LAWS (by Gustav Kirchhoff)

A Self-regulated Learning Module 77


 These laws are used to analyze and solve circuits that cannot be reduced by the rules of
combining resistors in series and in parallel, thus in this lesson we are going to analyze
complex circuits by applying Kirchhoff’s rules.

A. Current Law (KCL) or Junction Rule

 states that, “The sum of all currents entering a junction must be equal to the sum
of all currents leaving the junction”, or “The algebraic sum of all currents entering
and leaving a junction is equal to zero."

o ∑Iin=∑Iout

 A Junction or node is any point in a circuit where three. or more


electrical branches or wires are connected.

 applies to the charge entering and leaving a junction. Current is the flow of charge,
and charge is conserved; thus, whatever charge flows into the junction, must
flow out of it.

 Currents are indicated by arrows such that, arrows pointing toward the junction are entering
and currents pointing away from it are leaving the junction. Since we are taking the algebraic
sum, let us denote a positive current for those entering the junction and negative current for
those leaving the junction. And so we can rewrite the equation as,

I1+I2+I4+I5-I3-I6 = 0

B. Voltage Law (KVL) or Loop Rule

A Self-regulated Learning Module 78


 states that the algebraic sum of potential differences, including voltage supplied by
the voltage sources and resistive elements, in any loop must be equal to zero.

o V = 0

 Let us formulate an equation from the circuit below based on the loop rule,

 Starting from point a,


a. we move around the circuit in a clockwise direction, take note of the
voltage polarities (+ and – signs) as we move around passing through each
voltage source and the voltage drops
- polarity of the voltage source is fixed but that of the voltage drop will
depend on the direction of current and the direction of traverse or
motion of the loop

 When a resistor is traversed in the same direction as the


current, the change in potential is −IR.
 When a resistor is traversed in the direction opposite to the
current, the change in potential is +IR.
 When a voltage source is traversed from – to +, the change in
potential is +V.
 When an emf is traversed from + to –, the change in potential
is – V

V = -IR V = +IR

A Self-regulated Learning Module 79


V = +E V = -E

b. We can now formulate the loop equation starting from point a, to point b,
then to point c, though to point d and back to point a, we have

+V -I1R1 -I2R2 -I3R3 = 0, substituting the values we have,

12 -I1 -2I2 -3I3 = 0, and so we have an equation representing the loop


rule.

 Here are the steps to be done first before formulating the current and voltage
equations:

1. Label junction points in the circuit diagram using either lowercase or uppercase letters like a, b,
c, A, B, C so as not to confuse with the resistance labeling which are using numbers.
2. Locate the junctions in the circuit. The junctions are points where three or more wires connect.
Label each junction with the currents and directions into and out of it. Current directions are
assumed so you can assign any direction as you wish just make sure at least one current points
into the junction and at least one current points out of the junction.
3. Choose the loops in the circuit. Every component must be contained in at least one loop, but a
component may be contained in more than one loop.
4. Apply the junction rule and formulate the current equations.
5. Apply the loop rule and formulate the voltage equations.
 Note: Remember to only formulate equations equal to the number of unknowns,
so if you are solving for three resistor currents, then formulate only three
equations. Making more or less equations will just complicate your solution.

6. Solve for the unknown currents using rules of elimination and substitution which you have learned
in Algebra.
7. Check to see if you have arrived at the correct answers by substituting the values that you got to
the equations you formulated.

EXAMPLE:

Find the currents flowing thru each resistor in the given circuit.

A Self-regulated Learning Module 80


Solution:

1. We begin by applying Kirchhoff’s first or junction rule at point a. This gives

I1 = I2 + I3 – this is equation 1

- since I1 flows into the junction, while I2 and I3 flow out. Applying the junction rule at
e produces exactly the same equation, so that no new information is obtained.

* This is a single equation with three unknowns—three independent equations are


needed, and so the loop rule must be applied.

2. Now we consider the loop abcdea. Going from a to b, we traverse R 2 in the same direction
of the current I2, and so the change in potential is −I2R2. Then going from b to c, we go
from – to +, so that the change in potential is +E1. Traversing the internal resistance r 1
from c to d gives −I2r1. Completing the loop by going from d to a again traverses a resistor
in the same direction as its current, giving a change in potential of −I 1R1. The loop rule
states that the changes in potential sum to zero. Thus,
−I2R2 + E1 − I2r1 − I1R1 = −I2(R2 + r1) + emf1 − I1R1 = 0.

Substituting values from the circuit diagram for the resistances and emf, and
canceling the ampere unit gives

−3I2 + 18 − 6I1 = 0 – this is now equation 2

3. Now applying the loop rule to aefgha (we could have chosen abcdefgha as well) similarly
gives
+ I1R1 + I3R3 + I3r2 − V2 = + I1R1 + I3(R3 + r2) − V2 = 0.

Note that the signs are reversed compared with the other loop, because elements
are traversed in the opposite direction. With values entered, this becomes

A Self-regulated Learning Module 81


+6I1 + 2I3 − 45 = 0 – we have this as equation 3

4. These three equations are sufficient to solve for the three unknown currents.
I1 = I2 + I3

−3I2 + 18 − 6I1 = 0

+6I1 + 2I3 − 45 = 0

First, solve the second equation for I2:

I2 = 6 − 2I1

Now solve the third equation for I3:

I3 = 22.5 − 3I1

Substituting these two new equations into the first one allows us to find a value for I 1:

I1 = I2 + I3 = (6−2I1) + (22.5− 3I1) = 28.5 − 5I1.

Combining terms gives

6I1 = 28.5, and

I1 = 4.75 A

Substituting this value for I1 back into the fourth equation gives

I 2 = 6 − 2I1 = 6 − 9.50

I2 = −3.50 A
 The minus sign means I2 flows in the direction opposite to that assumed
in the circuit.
Finally, substituting the value for I1 into the fifth equation gives

I3 = 22.5 − 3I1 = 22.5 − 14 . 25

I3 = 8.25 A

5. Just as a check, we note that indeed I1 = I2 + I3. The results could also have been checked
by entering all of the values into the equation for the abcdefgha loop.

I1 = I2 + I3 = -3.50+8.25 = 4.75 A

 We can see that the value obtained is the same as the value solved
above for I1.

A Self-regulated Learning Module 82


Activity 12.3

PROBLEM SOLVING: Copy the questions in your formatted short bond papers and show your
complete and neat solutions. No erasures allowed.

 Find the currents flowing thru each resistor in the given circuit. Show step by step
solution. Substitute your answers to check the validity of the values that you arrived at.
Be patient and always double check your solution especially for the – and + signs of the
equations. 😊
 Point distribution:
Node and loop equations – 5 points each (15 points total)
Complete Solution – 10 points
Current values (I1, I2, I3, and I4) – 5 points each (15 points total)

SUMMARY

Let us now summarize the topics that we have learned from this module:
1. In a series circuit, all components are connected end-to-end, forming a single path for current
flow.
2. In a parallel circuit, all components are connected across each other, forming exactly two sets
of electrically common points.
3. A “branch” in a parallel circuit is a path for electric current formed by one of the load
components (such as a resistor).
4. Kirchhoff’s rules can be used to analyze any circuit, simple or complex.
5. The two rules are based, respectively, on the laws of conservation of charge and energy.
6. In actual electrical practice, consider safety at all times when dealing with electrical lines,
connections, outlets, and when using electrical equipment and devices.

ASSESSMENT (Quiz #8)

A Self-regulated Learning Module 83


 In this module, we will study the basic concepts on magnetism which plays many important roles
in our lives today. Physicists’ understanding of magnetism has enabled the development of
technologies that affect our everyday lives.

 The following are the lessons we will be taking up in this module:


LESSON 1 - Forces in magnetic fields
LESSON 2 - Sources of magnetic fields

LEARNING OUTCOMES:

 At the end of this module, the student should be able to:


1. Discuss competently the concepts on magnetism.
2. Describe electromagnetism and apply its principles to problem on magnetic field and torque.

LESSON 1: FORCES IN MAGNETIC FIELDS

A magnet is an object that exhibits magnetic properties such as,


- exerting an attractive force on iron or other ferromagnetic materials
- exerting both attractive and repulsive forces on other magnets
- deflecting the path of a moving charged particle

 Magnetic Poles
- Regions on a magnet where the forces are the strongest.
- Come in two types — north pole and south pole, often shortened to north and south or
abbreviated N and S.
- Magnetic poles always occur in north-south pairs called dipoles.

 Combinations of dipoles are called multipoles.


- Magnetic monopoles do not seem to exist.
- When a dipole magnet is broken, all of its pieces are also
dipoles.

 The magnetic field is the area around a magnet in which there is magnetic force.
 The magnetic field lines or forces are representations of the strength and the direction of a
magnetic field.

 Magnetic field is created by:


1. Moving electric charges and
2. current flowing through a wire.

A Self-regulated Learning Module 84


 These figures show the magnetic field
surrounding different shapes of magnets. The
magnetic field is represented by magnetic lines
of forces, which are shown here by lines and
arrows.
 Magnetic lines of forces comes out of the
north pole and goes into the south pole as
shown by the arrow directions

Magnetic field lines Magnetic field lines

between unlike poles between like poles

 The diagrams above show the behavior of the magnetic field lines of forces between unlike and
like poles of a magnet.

 The properties of magnetic field lines can be summarized by these rules:

1. The direction of the magnetic field is tangent to the field line at any point in space. A small compass
will point in the direction of the field line.
2. The strength of the field is proportional to the closeness of the lines. It is exactly proportional to
the number of lines per unit area perpendicular to the lines (called the magnetic filed strength or
density).
3. Magnetic field lines can never cross, meaning that the field is unique at any point in space.
4. Magnetic field lines are continuous, forming closed loops without beginning or end. They go from
the north pole to the south pole outside of the magnet, and from south pole to the north pole
inside of the magnet.
5. North and south poles cannot be separated, they always come together.

A. Magnetic field strength: Force on a moving charge in a magnetic field


- Magnetic fields exert forces on moving charges

 The magnitude of the magnetic force F on a charge q moving at a speed v in a magnetic


field of strength B is given by

F = qvB sin θ

A Self-regulated Learning Module 85


Where:

F – is the force in newton (N)


v – is the speed in m/s
q – is the charge of a proton/electron in coulomb (C) = ±1.6 x 10-19C
θ - is the angle between the directions of v and B

B - is the magnetic field strength in tesla (T), after the inventor Nikola Tesla
(1856-1943). Another smaller unit is called the gauss (G), equal to

1 G = 1 x 10-4 T or 1T = 1x104 G = 10,000 G

 This force is often called the Lorentz force. Lorentz force is the force experienced by a
particle of charge q moving with a velocity v in an electric field E and a magnetic field B.

 Since force is a vector quantity, then its direction should be indicated. How? By using the
concept of the Right-Hand-Rule (RHR).

The direction of the magnetic force F is perpendicular to the


plane formed by v and B, as determined by the right hand rule. Right-
hand-rule states that, to determine the direction of the magnetic force
on a positive moving charge, you point the thumb of your right hand in
the direction of v, the fingers in the direction of B, and a perpendicular
to the palm points in the direction of F as shown in the figure.

The thumb always points to the direction of velocity, extended


fingers point to the direction of magnetic field and the force is in the
direction you would push with your palm. The force on a negative charge is in exactly the opposite
direction to that on a positive charge.
 In the RHR, note that the three quantities namely, velocity, magnetic field and force, should be
perpendicular with each other, that is they are at right angle with each other.
 Watch the link below to have a better understanding of the Right-hand-rule:
- https://www.youtube.com/watch?v=PVignrHljXQ

EXAMPLE:

A proton moves at 8 x 106 m/s along the +x-axis and enters a region where there is a
magnetic field of magnitude 2.5 T, directed at an angle of 30 with the x-axis and lying in
the xy-plane. a) Find the magnitude and direction of the magnetic force on the proton.
b) If the proton were replaced with an electron, what will be the force magnitude and
direction acting on it?

A Self-regulated Learning Module 86


Y-axis
Given: By
B
30 V

Bx

v = 8 x 106 m/s

B = 2.5 T

θ = 30

q = 1.6 x 10-19C

Solution:

a) F = qvB sin θ = (1.6 x 10-19)( 8 x 106)(2.5)*sin 30 = 1.6 x 10-12 N, out of the
page

 Using RHR to determine the direction of the force, the velocity and the
magnetic field should be perpendicular with each other, so looking at the
diagram, notice that the component of the magnetic field that is perpendicular
with the velocity is By. Thus, placing our fingers in the direction of By which
is upward and the thumb in the direction of v, we can have the direction of
force which is toward you or outside of the page.

b) The force magnitude on an electron is the same as that of the proton, but its
direction is reversed, thus, F = 1.6 x 10-12 N, into the page

B. Magnetic Force on a Current-carrying Wire

- A charge moving through a magnetic field experiences a force due to the field.
- Since current is simply a stream of positive charges, a current experiences a force due
to a magnetic field.

Electromagnetism, is a branch of physics involving the study of the electromagnetic force, a


type of physical interaction that occurs between electrically charged particles. When electric
current flows through a simple wire, it creates a magnetic field in the whole conductor. The
magnetic field around the conductor has a definite direction North and south. This north and
south poles are determined the direction of the electric current flowing through the
conductor.

 A solenoid, comprising of a coil of wire, is an example of a device that


operates through the concept of electromagnetism, and is also an example of

A Self-regulated Learning Module 87


a temporary magnet, since magnetic field is only present when current flows
through it, but losses magnetism once current is removed.

 The magnitude of the magnetic force F on a length l of wire carrying current I in a magnetic
field of strength B is given by

F = IlB sin θ
Where:
F – is the force in newton (N)
l – is length of wire in meter (m)
I – is the current in ampere (A)
θ - is the angle between the directions of I and B
B - is the magnetic field strength in tesla (T)

 The direction of force can also be determined by using the Right-hand-rule (RHR).

The direction of the magnetic force F is perpendicular to the plane


formed by I and B, as determined by the right hand rule. Right-hand-rule
states that, to determine the direction of the magnetic force on current-
carrying wire, you point the thumb of your right hand in the direction of I,
the fingers in the direction of B, and a perpendicular to the palm points in
the direction of F as shown in the figure.

The thumb always points to the direction of current, extended fingers


point to the direction of magnetic field and the force is in the direction you would push with your palm.

EXAMPLE:
A wire carries a current of 22 A toward east. If the magnetic field in the area is vertical and
directed to the north with a magnitude of 0.5 x 10-4 T, find the magnitude and direction of
the magnetic force on a 36-m length of wire.

Given: B=0.5 x10-4T

I =22A l = 36 m
θ = 90, since B and I are perpendicular with each
other

Solution:

F = IlB sin θ = (22)(36)(0.5 x10-4) sin 90 = 0.0396 N, out of the page

A Self-regulated Learning Module 88


 The force direction as indicated by the RHR is out of the page, since if we
orient the thumb toward the East and the fingers toward the North, the palm
of our hands will be toward us.
3. Torque
- When a magnetic field exerts a force on a straight wire carrying current, it exerts a
torque on the loop of wire carrying current. Torque causes an object to spin around a
fixed axis.
- In equation,
=NIABsinθ
where:  - torque in N-m
N - turns or loops of wire
A – area of the coil in m2
I - current in A
B - magnetic field in Tesla
 - angle between the magnetic field and a perpendicular to the
plane of the coil

- Although the forces acting upon the loop are equal and opposite, they both act to rotate
the loop in the same direction.

 Example:
The 40-loop coil shown in the figure carries a current of 2A in a magnetic field of
B=0.25T. Find the torque on it. How will it rotate?

Solution:

A Self-regulated Learning Module 89


Activity 13

Problem solving: Copy the questions in your formatted short bond papers and show your
complete and neat solutions. No erasures allowed.
1. For the given figures a,b,c,d,e,f. ( 5 points each; 3 pts for magnitude, 2 points for direction)
Solve for the magnitude and direction of the force on a proton given the following particulars:

q = 1.6 x 10-19C

v = 10 x 106 m/s

B = 0.15 T

2. A 15 m long wire carries a current of 6 A and is placed in a uniform magnetic field of 0.40 T.
Solve for the magnitude and direction of the force on the wire when the wire is (a) perpendicular
to the field, and (b) at 30 to the field. Draw your diagram indicating the current and magnetic
field directions.
(5 points each for the drawing, 3 points for the magnitude, 2 points for the direction)

3. Cite two examples of devices using the concept of electromagnetism and explain how the
device works thru electromagnetism. (10 points)

A Self-regulated Learning Module 90


SUMMARY

Let us now summarize the topics that we have learned from this module:

1. Magnetic fields can be pictorially represented by magnetic field lines, the properties of
which are as follows:
a. The field is tangent to the magnetic field line.
b. Field strength is proportional to the line density.
c. Field lines cannot cross.
d. Field lines are continuous loops.

2. Magnetic fields exert a force on a moving charge q, the magnitude of which is

F=qvBsinθ

where θ is the angle between the directions of v and B

3. The SI unit for magnetic field strength B is the tesla (T)


4. The direction of the force on a moving charge is given by right hand rule (RHR):
- Point the thumb of the right hand in the direction of v, the fingers in the direction of B,
and a perpendicular to the palm points in the direction of F.
- The force is perpendicular to the plane formed by v and B. Since the force is zero if v is
parallel to B.

5. The magnetic force on current-carrying conductors is given by


F=IlBsinθ

where I is the current, l is the length of a straight conductor in a uniform


magnetic field B, and θ is the angle between I and B. The force follows RHR with
the thumb in the direction of I.

6. Magnetic fields are created by moving charges through conductors where current can pass
though and by materials possessing natural magnetism like bar magnets and the Earth.

ASSESSMENT (QUIZ #9)

A Self-regulated Learning Module 91


 In this module, we will study the basic concepts of light. Many phenomena in our everyday life
depend on the properties of light. The dual nature of light, which are reflection and refraction,
will be discussed briefly, as well as, explore the wave nature of light on optics.

 The following are the lessons we will be taking up in this module:

LESSON 1 - Reflection of Light

LESSON 2 - Refraction of Light

LEARNING OUTCOMES:

 At the end of this module, the student should be able to:


1. Define and discuss the basic concepts of light and optics
2. Apply these concepts by solving examples using step by step methods.
3. Solve problems correctly and confidently.

LESSON 1: REFLECTION OF LIGHT


 Light is basic to almost all life on Earth. It is the principal means by which we are able to transmit
and receive information to and from objects around us and throughout the Universe.
 Light is a form of electromagnetic radiation and represents energy transfer from the source to
the observer.
 When light is emitted or absorbed by the atoms of a system, these processes occur as if the
radiant energy is in the form of minute, localized, well-directed blasts; that is, as if light is a
stream of particles.

 Let us take a look at the one behavior of light which is REFLECTION.

A. What is reflection of Light?


- It is simply the bouncing back of light as it hits a surface.
- These figures show the behavior of light as it hits a reflective surface, like mirrors.

A Self-regulated Learning Module 92


B. Laws of Reflection

1. “The angle of incidence equals the angle of reflection.”


 The normal line, as shown in the diagram is drawn perpendicular (vertical line) to
the mirror surface.
 The reference of the angles of incidence and reflection is the normal line.

 If i is the angle of incidence and r


is the angle of reflection, then we can
write the relationship of the angles of
incidence and reflection as:

i = r

2. The incident ray, reflected ray and the normal line lie on the same surface, as shown in the
figures.

C. Plane and Spherical Mirrors


 Mirrors are smooth surfaces that reflects light at specific angles, forming an image of
the person or object in front of it.

1. Plane Mirror
- Are mirrors that form images which are virtual, upright, of the same size as the object and
as far behind the reflecting surface as the object is in front of it.
- Virtual images are images that are formed in locations where light does not actually reach.
Light does not actually pass through the location on the other side of the mirror; it only

A Self-regulated Learning Module 93


appears to an observer as though the light is coming from this location. Whenever a mirror
creates an image that is virtual, it will be located behind the mirror where light does not really
come from.

- Observe the candle in the figure as it is place in front of a plane mirror:

The image of a candle as viewed in a plane


mirror. The image has the same dimensions as the
object and is the same distance behind the mirror as
the object is in front of the mirror.

2. Spherical or Curved Mirrors


- A spherical mirror is a mirror that has the shape of a piece cut out of a spherical
surface.
- There are two types of spherical mirrors: concave and convex mirror.

 The following are basic terminologies that one needs to know while studying spherical
mirrors:
a. Center of Curvature
- The point in the center of the mirror that passes through the curve of the mirror and
has the same tangent and curvature at that point.
b. Radius of Curvature
- It’s the linear distance between Pole and the Center of curvature.
c. Principal axis
- The imaginary line passing through the optical center and the center of curvature of
any lens or a spherical mirror.
d. Pole
- The midpoint of the spherical mirror.
e. Aperture
- An aperture of a mirror or lens is a point from which the reflection of light actually
happens. It also gives the size of the mirror.
f. Principal Focus (real focus)
- Principal Focus can also be called Focal Point. It’s on the axis of a mirror or lens
wherein rays of light parallel to the axis converge after getting reflected from the
mirror.

A Self-regulated Learning Module 94


I. CONCAVE Mirror
 Are spherical mirrors that form inverted real images of objects placed beyond
the principal focus.

II. CONVEX Mirror


 Are spherical mirrors that produce upright virtual images of objects placed in
front of them and appear smaller in size.

III. The mirror equation expresses the quantitative relationship between the object
distance (do), the image distance (di), and the focal length (f). The equation is
stated as follows:
1 1 1
= +
𝑓 𝑑𝑜 𝑑𝑖

IV. The magnification equation (M) relates the ratio of the image distance and
object distance to the ratio of the image height (hi) and object height (ho). The
magnification equation is stated as follows:
ℎ𝑖 𝑑𝑖
M= ℎ𝑜
=- 𝑑𝑜

Example:
A 4-cm tall light bulb is placed a distance of 45.7 cm from a concave mirror
having a focal length of 15.2 cm. Determine the image distance (di) and the image size
(hi).
Given: ho = 4 cm
do = 45.7 cm

A Self-regulated Learning Module 95


f = 15.2 cm
Solution:
1 1 1 1 1 1 1
Solving for di, = + ; = + ; 0.0658 = 0.0219 +
𝑓 𝑑𝑜 𝑑𝑖 15.2 45.7 𝑑𝑖 𝑑𝑖

1
= 0.0658 – 0.0219 = 0.0439
𝑑𝑖
1
Thus, di = = 22.779 cm
0.0439

ℎ𝑖 𝑑𝑖 ℎ𝑖 22.779
Solving for hi, =- ; =-
ℎ𝑜 𝑑𝑜 4 45.7
22.779
Thus hi = 4(− 45.7
) = -1.994 cm

 The negative value for image height indicates that the image is an
inverted image.

 Watch the video link below to understand better the concepts on spherical mirrors.
 https://www.youtube.com/watch?v=aV5X9Ee7Rnc

Activity 14.1
A. ESSAY: Copy and the answer the following questions completely and neatly. (5 points each)
1. Using the law of reflection, explain how powder takes the shine off of a person’s nose.
2. You might have noticed that emergency vehicles such as ambulances are often labeled on
the front hood with reversed lettering (e.g., ECNALUBMA). Explain why this is so.

B. Problem solving: Copy the questions in your formatted short bond papers and show your
complete and neat solutions/answers. No erasures allowed.

 Determine the image distance and image height for a 5.00-cm tall object placed
45.0 cm from a concave mirror having a focal length of 15.0 cm. (10 points)
 Take a picture or scan your paper and upload in canvas.

LESSON 2: REFRACTION of LIGHT

- Refraction is the bending of light as it passes from one medium to another.


- A medium could be in the form of solid, liquid or air.
- The figure below shows the directions of the incident and refracted rays.

A Self-regulated Learning Module 96


A. Speed of Light (c)
- The bending of light is caused by the change in speed experienced by a wave when
it changes medium.
- The value of the speed of light in a vacuum is,
c = 2.9972458 × 108 m/s ≈ 3.00 × 108 m/s

B. Index of refraction ()


- We define the index of refraction  of a material to be the ratio of the speed of light
in a vacuum to that in the material, given as

𝒄
=
𝒗

Where: v - speed of light in the material


 Since the speed of light is always less than c in matter and equals c only in a
vacuum, the index of refraction is always greater than or equal to one.

C. Snell’s Law
- This law gives the relationship between the angles of incidence and refraction and
the indices of refraction of the two media and is given by,
n1 sinθ1 = n2 sinθ2

where: n1, n2 = are the indexes of refraction of the two media


1, 2 = are the angles of incidence and refraction, respectively

EXAMPLES:

1. What is the speed of light in water (n=1.333)? In glycerine (n=1.473)?


Solution:
𝑐
speed of light (v) for water = 
= 3 x 108/1.333 = 2.2506 x 108 m/s
𝑐
speed of light (v) for glycerine = 
= 3 x 108/1.473 = 2.0367 x 108 m/s

A Self-regulated Learning Module 97


2. A layer of oil (n=1.45) floats on water (n=1.33). A ray of light shines onto the oil with an
incidence angle of 40. Find the angle the ray makes in the water.

Given:

Solution:

 In the 1st equation, the noil sinoil is equated with the 2nd equation , giving the 3rd
equation being nair sin 40 = nwater sin water.

Activity 14.2

Problem solving: Copy the questions in your formatted short bond papers and show your
complete and neat solutions. No erasures allowed. Take a picture or scan your paper and upload
in canvas.

1. What is the speed of light in air (n=1.00293)? In crown glass (n=1.52)? (10 points)
2. A beam of light strikes the surface of water at an incidence angle of 60. Determine the angles
of the reflected and refracted rays. For water, n = 1.33. (10 points)

A Self-regulated Learning Module 98


SUMMARY

Let us now summarize the topics that we have learned from this module:

1. Reflection is the bouncing back of light waves while refraction is the bending of light waves.
2. Plane mirrors produce images with a number of distinguishable characteristics. Images formed
by plane mirrors are virtual, upright, left-right reversed, the same distance from the mirror as
the object's distance, and the same size as the object.
3. The Law of Reflection states that “the angle of reflection equals the angle of incidence”.
4. Spherical mirrors are classified as concave and convex mirrors.
5. The changing of a light ray’s direction when it passes through variations in matter is called
refraction.
6. Snell’s law, the Law of Refraction, is stated in equation form as n1 sin θ1 = n2 sinθ2.

ASSESSMENT (QUIZ #10)

A Self-regulated Learning Module 99


REFERENCES:

Bueche, F. J., & Hecht, E. (1997). Electrical Power, Equivalent Resistance; Simple Circuits, Kirchhoff’s
Laws. In Schaum's outline of theory and problems of college physics (p. 265-287). New York:
McGraw-Hill.

Bueche, F. J., & Hecht, E. (1997). Forces in Magnetic Fields, Sources of Magnetic Fields. In Schaum's
outline of theory and problems of college physics (p. 289-304). New York: McGraw-Hill.

Bueche, F. J., & Hecht, E. (1997). Potential;Capacitance, Inductance. In Schaum's outline of theory and
problems of college physics (p. 252, 253, 255, 321,323,327,328). New York: McGraw-Hill.

Bueche, F. J., & Hecht, E. (1997). Reflection and Refraction of Light, Interference and Diffraction of
Light. In Schaum's outline of theory and problems of college physics (p. 338-352, 366-367)). New
York: McGraw-Hill.

CamQue, R. (2016, November 03). IGCSE PHYSICS: Electromagnetic Effects. Retrieved July 23, 2020,
from https://www.slideshare.net/jamaerah/igcse-physics-electromagnetic-effects

Circuit Diagram. (n.d.). Retrieved July 22, 2020, from https://www.circuit-diagram.org/editor/

Equilibrium and Statics. (n.d.). Retrieved January 31, 2021, from


https://www.physicsclassroom.com/class/vectors/Lesson-3/Equilibrium-and-Statics

Freepik. (2019, February 26). Physics Science About The Movement Of Magnetic Fields. Retrieved July
22, 2020, from https://www.freepik.com/premium-vector/physics-science-about-movement-
magnetic-fields_4010627.htm

HyperPhysics. (n.d.). Magnetic Field. Retrieved July 23, 2020, from http://hyperphysics.phy-
astr.gsu.edu/hbase/magnetic/magfie.html

Itutor Follow. (2013, May 29). Light & optics. Retrieved July 25, 2020, from
https://www.slideshare.net/itutor/light-optics

J.M.K.C. Donev et al. (2020). Energy Education - Law of conservation of energy [Online]. Available:
https://energyeducation.ca/encyclopedia/Law_of_conservation_of_energy. [Accessed: July 24,
2020]

Jewett, J. W., & Serway, R. A. (2019). Current and Resistance. In Physics for Scientists and Engineers
with Modern Physics (10th ed., pp. 691-741). Singapore: Cenage Learning Asia Pte.

Jewett, J. W., & Serway, R. A. (2019). Light and Optics, Image Formation. In Physics for Scientists and
Wngineers with Modern Physics (10th ed., pp. 897-961). Singapore: Cenage Learning Asia Pte.

A Self-regulated Learning Module 100


Jewett, J. W., & Serway, R. A. (2019). Magnetic Fields. In Physics for Scientists and Engineers with
Modern Physics (10th ed., pp. 742-767). Singapore: Cenage Learning Asia Pte

Lettsome, C., Phd, PE (Director). (2015). How to Measure DC Voltage and Current in a Series Resistor
Circuit. [Video file]. Retrieved August 15, 2020, from
https://www.youtube.com/watch?v=6-NO3DR_sLo

Lettsome, C., Phd, PE (Director). (2015). How to Measure DC Voltage and Current in a Parallel Resistor
Circuit [Video file]. Retrieved August 15, 2020, from
https://www.youtube.com/watch?v=H8r7FRPOqEE&amp;feature=youtu.be

Mason, J. (Director). (2013). How to Solve a Kirchhoff's Rules Problem - Simple Example [Video file].
Retrieved July 22, 2020, from https://www.youtube.com/watch?v=Z2QDXjG2ynU

Mason, J. (Director). (2015, March 31). How to Solve Any Series and Parallel Circuit Problem [Video
file]. Retrieved July 22, 2020, from https://www.youtube.com/watch?v=-PiB2Xd3P94

National MagLab (Director). (2017, February 28). How Electromotive Force Works [Video file].
Retrieved August 16, 2020, from https://www.youtube.com/watch?v=shJAV59NS6k&t=40s

Olsson, F. (Director). (2014). Setting up Ohm's Law circuit [Video file]. Retrieved July 22, 2020, from
https://www.youtube.com/watch?v=Vh3XGz7hgU4

OpenStax. (n.d.). Physics. Retrieved July 22, 2020, from


https://courses.lumenlearning.com/physics/chapter/21-3-kirchhoffs-rules/

OpenStax. (n.d.). Physics. Retrieved July 22, 2020, from


https://courses.lumenlearning.com/physics/chapter/22-3-magnetic-fields-and-magnetic-field-
lines/

OpenStax. (n.d.). Physics. Retrieved July 25, 2020, from


https://courses.lumenlearning.com/physics/chapter/25-2-the-law-of-reflection/

Physics Tutorial. (n.d.). Physics Tutorial: Image Characteristics of Plane Mirrors. Retrieved July 25,
2020, from https://www.physicsclassroom.com/class/refln/Lesson-2/Image-Characteristics

Robinson, E. (2015, July 04). AS Level Physics- Electric circuits. Retrieved July 22, 2020, from
https://www.slideshare.net/ebinrobinson/as-level-physics-electric-circuits

Stark, G. (2020, March 24). Characteristics of waves. Retrieved July 26, 2020, from
https://www.britannica.com/science/light/Characteristics-of-waves

The Physics Hypertexbook. (n.d.). Retrieved January 31, 2021, from https://physics.info/

A Self-regulated Learning Module 101


Tutorialspoint. (2020). Physics - Electricity. Retrieved July 22, 2020, from
https://www.tutorialspoint.com/physics_part1/physics_electricity.htm

Vectors. (n.d.). Retrieved January 31, 2021, from https://courses.lumenlearning.com/boundless-


physics/chapter/vectors/

A Self-regulated Learning Module 102


Please answer the following questions objectively.

a. What lesson or activity did I enjoy most? Why?

_____________________________________________________________________________
_____________________________________________________________________________
_____________________________________________________________________________

b. What is the most important lesson which I can apply in my daily life?

_____________________________________________________________________________
_____________________________________________________________________________
_____________________________________________________________________________

c. What are the new insights/discoveries that I learned?

_____________________________________________________________________________
_____________________________________________________________________________
_____________________________________________________________________________

d. What topics do I find least important?

_____________________________________________________________________________
_____________________________________________________________________________
_____________________________________________________________________________

e. What possible topics should have been included?

_____________________________________________________________________________
_____________________________________________________________________________
_____________________________________________________________________________

THANK YOU VERY MUCH!!! KEEP GOING AND GOD BLESS YOU ON YOUR NEXT ENDEAVOR 😊

A Self-regulated Learning Module 103


A Self-regulated Learning Module 104

You might also like